A 25 year old man is shot in the abdomen and is transferred to the operating theatre following arrival in the emergency department, as he is unstable and a FAST scan is positive. At operation there is an extensive laceration to the right lobe of the liver and involvement of the IVC. There is massive haemorrhage. What is the most appropriate approach to blood component therapy?

Use Factor VIII concentrates early

Avoid use of "o" negative blood

Transfuse packed cells, FFP and platelets in fixed ratios of 1:1:1

Transfuse packed cells and FFP in a fixed ratio of 4:1

Perform goal directed transfusion based on the Hb, PT and TEG studies

There is strong evidence to support the use of haemostatic transfusion in the setting of major haemorrhage due to trauma. This advocates the use of 1:1:1 ratios. Please rate this question:

Discuss and give feedback Next question Transfusions in

Uncontrolled haemorrhage accounts for up to 39% of all trauma related death. In the UK approximately 2% of all trauma patients will need massive transfusion.Massive transfusion is defined as the replacement of a patient's total blood volume in less than 24 hours, or as the acute administration of more than half the patient's estimated blood volume per hour. In haemorrhaging patients following trauma there is evidence to support the initial administration of tranexamic acid (CRASH study). During acute bleeding the practice of haemostatic has been shown to reduce mortality rates. The principle of haemostatic resuscitation is that blood components are transfused in fixed ratios. For example; packed red cells, FFP and platelets are administered in a ratio of 1:1:1.

The typical therapeutic end points include:

 Hb: 8-10 g/dl  Platelets > 100  PT and APTT (INR)< 1.5  Fibrinogen > 1.0 g/l  Ca2+ > 1 mmol/l  pH: 7.35-7.45  BE: +/- 2  ToC > 36 °C

Next question

A 76 year old woman with a body weight of 50 kg is undergoing an excision of a lipoma from her forehead. It is the first time the senior house officer has performed the procedure. He administers 30ml of 2% lignocaine to the area. The procedure is complicated by bleeding and the patient experiences discomfort, a further 10ml of the same anaesthetic formulation is then administered. Over the following 5 minutes the patient complains of tinnitus and becomes drowsy. Which of the drugs listed below should be administered?

Temazepam

Lorazepam

Naloxone

Intralipid 20%

Sodium bicarbonate 20%

Local anaesthetic toxicity treatment = Intralipid

Intralipid is indicated for the treatment of local anaesthetic toxicity. In this case the safe dose of local anaesthetic has been exceeded and is thus this lady's symptoms are likely to represent toxicity. Please rate this question:

Discuss and give feedback Next question Local anaesthetic toxicity

Toxicity results from either accidental intravascular injection (rapid onset of symptoms-usually correct dose), or from excessive dosage (slower onset). Local anaesthetic agents not only exert a membrane stabilising effect on peripheral nerves but will also act on excitable membranes within the CNS and Heart. The sensory neurones in the CNS are suppressed before the motor ones. As a result the early symptoms will typically be those of circumoral paraesthesia and tinnitus, followed by falling GCS and eventually coma.

Management of toxicity

 Stop injecting the anaesthetic agent  High flow 100% oxygen via face mask  Cardiovascular monitoring  Administer lipid emulsion (Intralipid 20%) at 1.5ml/Kg over 1 minute as a bolus  Consider lipid emulsion infusion, at 0.25ml/ Kg/ minute  If toxicity due to prilocaine then administer methylene blue

Safe doses 10ml of lignocaine 1% contains 100mg of drug, this would constitute 70% of the maximum safe dose in a 50 kg patient. Up to 7mg / kg can be administered if adrenaline is added to the solution.

Doses of local anaesthetics Agent Dose plain Dose with adrenaline

Lignocaine 3mg/Kg 7mg/Kg

Bupivicane 2mg/Kg 2mg/Kg

Prilocaine 6mg/Kg 9mg/Kg

These are a guide only as actual doses depend on site of administration, tissue vascularity and co- morbidities. Next question

A 27 year old man is involved in a road traffic accident. He is seen in the emergency department with chest pain. Clinical examination is essentially unremarkable and he is discharged. He subsequently is found dead at home. What is the most likely underlying ?

Tracheobronchial tree injury

Traumatic aortic disruption

Cardiac laceration

Diaphragmatic rupture

Rupture of the oesophagus

Theme from January 2013 Exam Aortic that do not die at the scene may have a contained haematoma. Clinical signs are subtle and the diagnosis may not be apparent on clinical examination. Without prompt treatment the haematoma usually bursts and the patient dies. Please rate this question:

Discuss and give feedback Next question Thoracic aorta rupture

 Mechanism of injury: Decelerating force i.e. RTA, fall from a great height  Most people die at scene  Survivors may have an incomplete laceration at the ligamentum arteriosum of the aorta.

Clinical features

 Contained haematoma: persistent hypotension  Detected mainly by history, CXR changes

CXR changes

 Widened mediastinum  Trachea/Oesophagus to right  Depression of left main stem bronchus  Widened paratracheal stripe/paraspinal interfaces  Space between aorta and pulmonary artery obliterated  Rib fracture/left haemothorax

Diagnosis Angiography, usually CT aortogram.

Treatment Repair or replacement. Ideally they should undergo endovascular repair. Next question

A patient is brought to the emergency department following a motor vehicle accident. He is unconscious and has a deep scalp laceration. His heart rate is 120/min, blood pressure is 80/40 mmHg, and respiratory rate is 35/min. Despite rapid administration of 2 litres of Hartmans solution, the patient's vital signs do not change significantly. The injury likely to explain this patient's hypotension is:

Epidural haematoma

Sub dural haematoma

Intra parenchymal brain haemorrhage

Base of skull fracture

None of the above

Raised intracranial pressure (Cushing response)

 Hypertension  Bradycardia  Respiratory depression

In the patient described, hypotension and tachycardia should not be uncritically attributed to the , since these findings in the setting of are suggestive of serious thoracic, abdominal, or pelvic hemorrhage. When cardiovascular collapse occurs as a result of rising intracranial pressure, it is generally accompanied by hypertension, bradycardia, and respiratory depression. Please rate this question:

Discuss and give feedback Next question Head injury

Patients who suffer head injuries should be managed according to ATLS principles and extra cranial injuries should be managed alongside cranial trauma. Inadequate cardiac output will compromise CNS perfusion irrespective of the nature of the cranial injury.

Types of

Extradural Bleeding into the space between the dura mater and the skull. Often results from haematoma acceleration-deceleration trauma or a blow to the side of the head. The majority of extradural haematomas occur in the temporal region where skull fractures cause a rupture of the middle meningeal artery.

Features

 Raised intracranial pressure  Some patients may exhibit a lucid interval

Bleeding into the outermost meningeal layer. Most commonly occur around the frontal and parietal lobes. May be either acute or chronic. Subdural haematoma Risk factors include old age and alcoholism.

Slower onset of symptoms than a extradural haematoma.

Usually occurs spontaneously in the context of a ruptured cerebral aneurysm, but Subarachnoid may be seen in association with other injuries when a patient has sustained a haemorrhage traumatic brain injury.

Pathophysiology

 Primary brain injury may be focal (contusion/ haematoma) or diffuse (diffuse axonal injury)  Diffuse axonal injury occurs as a result of mechanical shearing following deceleration, causing disruption and tearing of axons  Intra-cranial haematomas can be extradural, subdural or intracerebral, while contusions may occur adjacent to (coup) or contralateral (contre-coup) to the side of impact  Secondary brain injury occurs when cerebral oedema, ischaemia, infection, tonsillar or tentorial herniation exacerbates the original injury. The normal cerebral auto regulatory processes are disrupted following trauma rendering the brain more susceptible to blood flow changes and hypoxia  The Cushings reflex (hypertension and bradycardia) often occurs late and is usually a pre terminal event

Management

 Where there is life threatening rising ICP such as in extra dural haematoma and whilst theatre is prepared or transfer arranged use of IV mannitol/ frusemide may be required.  Diffuse cerebral oedema may require decompressive craniotomy  Exploratory Burr Holes have little management in modern practice except where scanning may be unavailable and to thus facilitate creation of formal craniotomy flap  Depressed skull fractures that are open require formal surgical reduction and debridement, closed injuries may be managed non operatively if there is minimal displacement.  ICP monitoring is appropriate in those who have GCS 3-8 and normal CT scan.  ICP monitoring is mandatory in those who have GCS 3-8 and abnormal CT scan.  Hyponatraemia is most likely to be due to syndrome of inappropriate ADH secretion.  Minimum of cerebral perfusion pressure of 70mmHg in adults.  Minimum cerebral perfusion pressure of between 40 and 70 mmHg in children.

Interpretation of pupillary findings in head injuries Pupil size Light response Interpretation

Unilaterally dilated Sluggish or fixed 3rd nerve compression secondary to tentorial herniation

Bilaterally dilated Sluggish or fixed  Poor CNS perfusion  Bilateral 3rd nerve palsy

Unilaterally dilated or Cross reactive (Marcus - Optic nerve injury equal Gunn)

Bilaterally constricted May be difficult to  Opiates assess  Pontine lesions  Metabolic encephalopathy

Unilaterally Preserved Sympathetic pathway disruption constricted Next question

A 42 year old man is admitted to surgery with acute appendicitis. He is known to have hypertension, psoriatic arthropathy and polymyalgia rheumatica. His medical therapy includes: Paracetamol 1g qds Codeine phosphate 30mg qds Bendrofluazide 2.5 mg od Ramipril 10mg od Methotrexate 7.5mg once a week Prednisolone 5mg od You are called by the Senior House Officer to assess this man as he has become delirious and hypotensive 2 hours after surgery. His blood results reveal:

Na+ 132 mmol/l

K+ 5.2 mmol/l

Urea 10 mmol/l

Creatinine 111 µmol/l

Glucose 3.5

CRP 158

Hb 10.2 g/dl

Platelets 156 * 109/l

WBC 14 * 109/l

What is the most likely diagnosis?

Septic shock secondary to appendicitis

Neutropenic sepsis

Phaeochromocytoma

Perforated bowel

Addisonian crisis

Features of an addisonian crisis:

 Hyponatraemia  Hyperkalaemia  Hypoglycaemia

This man is on steroids for polymyalgia rheumatica. Surgery can precipitate acute adrenal deficiency. The diagnosis is further confirmed by the blood results of hyponatraemia, hyperkalaemia and hypoglycaemia. This patient urgently needs hydrocortisone. Please rate this question:

Discuss and give feedback Next question Addisonian crisis

Causes

 Sepsis or surgery causing an acute exacerbation of chronic insufficiency (Addison's, Hypopituitarism)  Adrenal haemorrhage eg Waterhouse-Friderichsen syndrome (fulminant meningococcemia)  Steroid withdrawal

Management

 Hydrocortisone 100 mg im or iv  1 litre normal saline infused over 30-60 mins or with dextrose if hypoglycaemic  Continue hydrocortisone 6 hourly until the patient is stable. No fludrocortisone is required because high cortisol exerts weak mineralocorticoid action  Oral replacement may begin after 24 hours and be reduced to maintenance over 3-4 days

Next question

A 32 year old man is involved in a motorcycle accident and sustains a closed unstable spiral tibial fracture. This is managed with an intramedullary nail. On return to the ward he is noted to have increasing pain in the limb and on examination the limb is swollen and tender with pain on passive stretching of the toes. The most likely diagnosis is:

Tibial nerve neuropraxia

Displaced tibial nail

Compartment syndrome

Deep vein thrombosis

Sciatic nerve injury

Theme from September 2014 Exam Severe pain in a limb should raise suspicions of especially in tibial fractures following fixation with intra medullary devices. Please rate this question:

Discuss and give feedback Next question Compartment syndrome

 This is a particular complication that may occur following fractures (or following ischaemia re- perfusion injury in vascular patients). It is characterised by raised pressure within a closed anatomical space.  The raised pressure within the compartment will eventually compromise tissue perfusion resulting in necrosis. The two main fractures carrying this complication include supracondylar fractures and tibial shaft injuries.

Symptoms and signs

 Pain, especially on movement (even passive)  Parasthesiae  Pallor may be present  Arterial pulsation may still be felt as the necrosis occurs as a result of microvascular compromise  Paralysis of the muscle group may occur

Diagnosis

 Is made by measurement of intracompartmental pressure measurements. Pressures in excess of 20mmHg are abnormal and >40mmHg is diagnostic.

Treatment

 This is essentially prompt and extensive fasciotomies  In the lower limb the deep muscles may be inadequately decompressed by the inexperienced operator when smaller incisions are performed  Myoglobinuria may occur following fasciotomy and result in renal failure and for this reason these patients require aggressive IV fluids  Where muscle groups are frankly necrotic at fasciotomy they should be debrided and amputation may have to be considered  Death of muscle groups may occur within 4-6 hours

Next question

A 28 year old man is involved in a road traffic accident and sustains a injury. On arrival in the emergency department he is hypotensive. On examination; he has an elevated jugular venous pulse and auscultation of the heart reveals quiet heard sounds. What is the most likely diagnosis?

Pneumothorax

Myocardial contusion

Cardiac tamponade

Haemothorax

Ventricular septal defect

Theme from 2010 exam The presence of a is suggested by Becks Triad:

 Hypotension  Muffled heart sounds  Raised JVP

Please rate this question:

Discuss and give feedback Next question Thoracic trauma

Types of thoracic trauma

Tension  Often laceration to lung parenchyma with flap  Pressure develops in thorax  Most common cause is mechanical ventilation in patient with pleural injury  Symptoms overlap with cardiac tamponade, hyper-resonant percussion note is more likely in tension pnemothorax

Flail chest  Chest wall disconnects from thoracic cage  Multiple rib fractures (at least two fractures per rib in at least two ribs)  Associated with  Abnormal chest motion  Avoid over hydration and fluid overload

Pneumothorax  Most common cause is lung laceration with air leakage  Most traumatic pneumothoraces should have a chest drain  Patients with traumatic pneumothorax should never be mechanically ventilated until a chest drain is inserted

Haemothorax  Most commonly due to laceration of lung, intercostal vessel or internal mammary artery  Haemothoraces large enough to appear on CXR are treated with large bore chest drain  Surgical exploration is warranted if >1500ml blood drained immediately

Cardiac tamponade  Beck's triad: elevated venous pressure, reduced arterial pressure, reduced heart sounds  Pulsus paradoxus  May occur with as little as 100ml blood

Pulmonary contusion  Most common potentially lethal  Arterial blood gases and pulse oximetry important  Early intubation within an hour if significant hypoxia

Blunt cardiac injury  Usually occurs secondary to chest wall injury  ECG may show features of myocardial infarction  Sequelae: hypotension, arrhythmias, cardiac wall motion abnormalities

Aorta disruption  Deceleration injuries  Contained haematoma  Widened mediastinum

Diaphragm  Most due to motor vehicle accidents and blunt trauma causing disruption large radial tears (laceration injuries result in small tears)  More common on left side  Insert gastric tube, may pass into intrathoracic stomach

Mediastinal  Entrance in one hemithorax and exit wound/foreign body traversing in opposite hemithorax  Mediastinal haematoma or pleural cap suggests great vessel injury  Mortality is 20%

References ATLS Manual 8th Edition Next question

A 31 year old lady is struck by a car and is 32 weeks pregnant. On arrival in the emergency department she has a systolic blood pressure of 105mmHg and a pulse rate of 126 beats per minute. Abdominal examination demonstrates a diffusely tender abdomen and some left sided flank bruising. A FAST scan is normal. What is the most appropriate course of action?

Arrange a departmental abdominal USS scan

Arrange an urgent abdominal MRI scan

Perform a laparotomy

Perform diagnostic peritoneal lavage

Arrange an urgent abdominal CT scan

The patient's mechanism of injury makes a solid organ injury likely. FAST scanning is associated with a false negative rate in pregnancy which makes the negative result less reassuring. CT scanning remains the gold standard.

Please rate this question:

Discuss and give feedback

Next question

Imaging in the pregnant trauma patient

Sonography and FAST scanning are established in pregnancy and have the advantage of avoiding ionising radiation. However, the sensitivity of the FAST scan is reduced in pregnancy especially with advanced gestational age. Sensitivity of FAST scanning is 60-80% across all trimesters and 90% in the first. CT scanning remains the first line investigation in major trauma where significant visceral injury is suspected. The maximum permitted safe dose of radiation in pregnancy is 5mSv. A pelvic CT scan would fall below this level. That said, early exposure to radiation will increase the risk of developmental anomalies and foetal loss. Late exposure increases the risk of childhood cancer twofold. CT scanning remains the most sensitive test for identifying complications such as placental abruption in this group.

Next question

Theme: Trauma

A. Tension pneumothorax B. Haemopericardium C. Haemothorax D. Aortic transection E. Ruptured spleen F. Duodeno-jejunal flexure disruption G. Aorto iliac disruption H. Ileo-colic junction disruption

For each scenario please select the most likely injury. Each option may be used once, more than once or not at all.

9. A 24 year old motorist is involved in a road traffic accident in which he collides with the wall of a tunnel in a head on car crash, speed 85mph. He is wearing a seatbelt and the airbags have deployed. When rescuers arrive he is lucid and conscious and then dies suddenly.

You answered Tension pneumothorax

The correct answer is Aortic transection

Aortic transections typically occur distal to the ligamentum arteriosum. A temporary haematoma may prevent the immediate death that usually occurs. This is a deceleration injury. A widened mediastinum may be seen on x-ray.

10. A 30 year old women is involved in a road traffic accident she is a passenger in a car involved in a head on collision with another vehicle. Her car is travelling at 60mph. She has been haemodynamically stable throughout with only minimal tachycardia. On examination she has marked abdominal tenderness and a large amount of intra abdominal fluid on CT scan

You answered Tension pneumothorax

The correct answer is Duodeno-jejunal flexure disruption

This is another site of sudden deceleration injury. Given the large amount of free fluid, if it were blood, then a greater degree of haemodynamic instability would be expected.

11. A 17 year old boy is involved in a motorcycle accident in which he is thrown from his motorcycle. On admission he has distended neck veins and a weak pulse. The trachea is central.

You answered Tension pneumothorax

The correct answer is Haemopericardium

This is most likely a cardiac tamponade produced by haemopericardium. As little as 100ml of blood may result in tamponade as the pericardial sac is not distensible. Diagnosis is suggested by muffled heart sounds, paradoxical pulse and jugular vein distension.

Please rate this question:

Discuss and give feedback Next question Thoracic trauma

Types of thoracic trauma

Tension  Often laceration to lung parenchyma with flap pneumothorax  Pressure develops in thorax  Most common cause is mechanical ventilation in patient with pleural injury  Symptoms overlap with cardiac tamponade, hyper-resonant percussion note is more likely in tension pnemothorax

Flail chest  Chest wall disconnects from thoracic cage  Multiple rib fractures (at least two fractures per rib in at least two ribs)  Associated with pulmonary contusion  Abnormal chest motion  Avoid over hydration and fluid overload

Pneumothorax  Most common cause is lung laceration with air leakage  Most traumatic pneumothoraces should have a chest drain  Patients with traumatic pneumothorax should never be mechanically ventilated until a chest drain is inserted

Haemothorax  Most commonly due to laceration of lung, intercostal vessel or internal mammary artery  Haemothoraces large enough to appear on CXR are treated with large bore chest drain  Surgical exploration is warranted if >1500ml blood drained immediately

Cardiac tamponade  Beck's triad: elevated venous pressure, reduced arterial pressure, reduced heart sounds  Pulsus paradoxus  May occur with as little as 100ml blood

Pulmonary contusion  Most common potentially lethal chest injury  Arterial blood gases and pulse oximetry important  Early intubation within an hour if significant hypoxia

Blunt cardiac injury  Usually occurs secondary to chest wall injury  ECG may show features of myocardial infarction  Sequelae: hypotension, arrhythmias, cardiac wall motion abnormalities

Aorta disruption  Deceleration injuries  Contained haematoma  Widened mediastinum

Diaphragm  Most due to motor vehicle accidents and blunt trauma causing disruption large radial tears (laceration injuries result in small tears)  More common on left side  Insert gastric tube, may pass into intrathoracic stomach

Mediastinal  Entrance wound in one hemithorax and exit wound/foreign body traversing wounds in opposite hemithorax  Mediastinal haematoma or pleural cap suggests great vessel injury  Mortality is 20%

References ATLS Manual 8th Edition Next question

A 14-year-old boy is admitted to the acute surgical unit with appendicitis. He is normally fit and well. Apart from metoclopramide, the patient has had no other medications. The nursing staff contact you as the patient is acting strange. On examination he is agitated, has a clenched jaw and his eyes are deviated upwards. What is the most likely diagnosis?

Functional disorder

Malignant hyperthermia

Oculogyric crisis

Epilepsy

Serotonin syndrome

This is a classic description of an oculogyric crisis, a form of extrapyramidal disorder. An oculogyric crisis is an acute dystonic reaction. This is precipitated by antipsychotics (haloperidol) and metoclopramide in susceptible individuals with a genetic predisposition to this. Treatment is with procyclidine IM. Please rate this question:

Discuss and give feedback Next question Oculogyric crisis

An oculogyric crisis is a dystonic reaction to certain drugs or medical conditions

Features

 Restlessness, agitation  Involuntary upward deviation of the eyes

Causes

 Phenothiazines  Haloperidol  Metoclopramide  Postencephalitic Parkinson's disease

Management

 Procyclidine

Next question

A 6 year old boy pulls over a kettle and suffers superficial partial thickness to his legs. Which of the following will not occur?

Preservation of hair follicles

Formation of vesicles or bullae

Damage to sweat glands

Healing by re-epithelialisation

Pain at the site

Partial thickness burns are divided into superficial and deep burns, however, this is often not possible on initial assessment and it may be a week or more before the distinction is clear cut. Dermal appendages are, by definition, intact. Superficial partial thickness burns will typically heal by re-epithelialisation, deeper burns will heal with scarring. Please rate this question:

Discuss and give feedback Next question Burns

Burns may be thermal, chemical or electrical. In the former category are burns which occur as a result of heat. Chemical burns occur when the skin is exposed to an extremely caustic or alkaline substance. Electrical burns occur following exposure to electrical current. The immediate management includes removal of the burning source which usually includes irrigation of the burned area. A detailed assessment then needs to be made of the extent of the burns and a number of charts are available for recording this information. The degree of injury relates to the temperature and duration of exposure. Most domestic burns are mainly scalds in young children.

Following the burn, there is a local response with progressive tissue loss and release of inflammatory cytokines. Systemically, there are cardiovascular effects resulting from fluid loss and sequestration of fluid into the third space. There is a marked catabolic response. Immunosupression is common with large burns and bacterial translocation from the gut lumen is a recognised event. Sepsis is a common cause of death following major burns.

Types of burn

Type of burn Skin layers Skin Blanching Management affected appearance

Epidermal/Superficial Epidermis Red, moist Yes

Superficial partial Epidermis and part Pale, dry Yes Normally heals thickness of papillary dermis with no affected intervention

Deep partial thickness Epidermis, whole Mottled red No Needs surgical papillary dermis colour intervention affected (depending on site)

Full thickness Whole skin layer Dry, leathery No Burns centre and subcutaneous hard wound tissue affected

Depth of burn assessment

 Bleeding on needle prick  Sensation  Appearance  Blanching to pressure

Percentage burn estimation Lund Browder chart: most accurate even in children Wallace rule of nines Palmar surface: surface area palm = 0.8% burn

>15% body surface area burns in adults needs urgent burn fluid resuscitation

Transfer to burn centre if:

 Need burn shock resuscitation  Face/hands/genitals affected  Deep partial thickness or full thickness burns  Significant electrical/chemical burns

Management The initial aim is to stop the burning process and resuscitate the patient. Intravenous fluids will be required for children with burns greater than 10% of total body surface area. Adults with burns greater than 15% of total body surface area will also require IV fluids. The fluids are calculated using the Parkland formula which is; volume of fluid= total body surface area of the burn % x weight (Kg) x4. Half of the fluid is administered in the first 8 hours. A urinary catheter should be inserted. Analgesia should be given. Complex burns, burns involving the hand perineum and face and burns >10% in adults and >5% in children should be transferred to a burns unit.

Circumferential burns affecting a limb or severe torso burns impeding respiration may require escharotomy to divide the burnt tissue.

Conservative management is appropriate for superficial burns and mixed superficial burns that will heal in 2 weeks. More complex burns may require excision and skin grafting. Excision and primary closure is not generally practised as there is a high risk of infection.

There is no evidence to support the use of anti microbial prophylaxis or topical antibiotics in burn patients.

Escharotomies

 Indicated in circumferential full thickness burns to the torso or limbs.  Careful division of the encasing band of burn tissue will potentially improve ventilation (if the burn involves the torso), or relieve compartment syndrome and oedema (where a limb is involved)

References www.euroburn.org/e107files/downloads/guidelinesburncare.pdf

Barajas-Nava LA, López-Alcalde J, Roqué i Figuls M, Solà I, Bonfill Cosp X. Antibiotic prophylaxis for preventing burn wound infection. Cochrane Database of Systematic Reviews 2013, Issue 6. Art. No.: CD008738. DOI: 10.1002/14651858.CD008738.pub2.

Hettiaratchy S & Papini R. Initial management of a major burn: assessment and resuscitation. BMJ 2004;329:101-103 Next question

You are called to the acute surgical unit. A patient who has short gut syndrome has developed a broad complex tachycardia. You suspect a diagnosis of ventricular tachycardia. What is the most likely precipitant?

Hypoglycaemia

Bisoprolol

Hypomagnesaemia

Dehydration

Hyperthyroidism

Please rate this question:

Discuss and give feedback Next question Ventricular tachcardia

Ventricular tachycardia (VT)is broad-complex tachycardia originating from a ventricular ectopic focus. It has the potential to precipitate ventricular fibrillation and hence requires urgent treatment.

There are two main types of VT:

 monomorphic VT: most commonly caused by myocardial infarction  polymorphic VT: A subtype of polymorphic VT is torsades de pointes which is precipitated by prolongation of the QT interval. The causes of a long QT interval are listed below

Causes of a prolonged QT interval

Congenital Drugs Other

 Jervell-Lange-Nielsen  amiodarone, sotalol,  electrolyte: syndrome (includes class 1a hypocalcaemia, deafness and is due to an antiarrhythmic drugs hypokalaemia, abnormal potassium  tricyclic hypomagnesaemia channel) antidepressants,  acute myocardial  Romano-Ward syndrome fluoxetine infarction (no deafness)  chloroquine  myocarditis  terfenadine  hypothermia  erythromycin  subarachnoid haemorrhage

Next question

Based on the current guidelines, which option regarding management of head injuries is false?

Opiates should be avoided

Consider intubation if the GCS is <8 or = 8

Immediate CT head if there is > 1 episode of vomiting

Half hourly GCS assessment until GCS is 15

Contact neurosurgeons if suspected penetrating injury

Once eight, intubate!

Pain should be controlled, with opiates preferably, as this avoids distress and hypertension post injury. Please rate this question:

Discuss and give feedback Next question Head injury management- NICE Guidelines

Summary of guidelines

 All patients should be assessed within 15 minutes on arrival to A&E  Document all 3 components of the GCS  If GCS <8 or = to 8, consider stabilising the airway  Treat pain with low dose IV opiates (if safe)  Full spine immobilisation until assessment if:

- GCS < 15 - neck pain/tenderness - paraesthesia extremities - focal neurological deficit - suspected c-spine injury

If a c-spine injury is suspected a 3 view c-spine x-ray is indicated. CT c-spine is preferred if: - Intubated - GCS <13 - Normal x-ray but continued concerns regarding c-spine injury - Any focal neurology - A CT head scan is being performed - Initial plain films are abnormal

Immediate CT head (within 1 hour) if:

 GCS < 13 on admission  GCS < 15 2 hours after admission  Suspected open or depressed skull fracture  Suspected skull base fracture (panda eyes, Battle's sign, CSF from nose/ear, bleeding ear)  Focal neurology  Vomiting > 1 episode  Post traumatic seizure  Coagulopathy

Contact neurosurgeon if:

 Persistent GCS < 8 or = 8  Unexplained confusion > 4h  Reduced GCS after admission  Progressive neurological signs  Incomplete recovery post seizure  Penetrating injury  Cerebrospinal fluid leak

Observations

 1/2 hourly GCS until 15

Reference 1. http://guidance.nice.org.uk/CG56/QuickRefGuide/pdf/English 2. Hodgkinson S et al. Early management of head injury: summary of NICE guidance. BMJ 2014 (348):34-37. Next question

A 22 year old man suffers 20% partial and full thickness burns in a house fire. There is an associated inhalational injury. It is decided to administer intravenous fluids to replace fluid losses. Which of the intravenous fluids listed below should be used for initial resuscitation?

Dextran 40

5% Dextrose

Fresh frozen plasma

Hartmans solution

Blood

In most units a crystalloid such as Hartmans (Ringers lactate) is administered initially. Controversy does remain and some units do prefer colloid. Should this leak in the interstial tissues this may increase the risk of oedema. Please rate this question:

Discuss and give feedback Next question Fluid resuscitation burns

Indication: >15% total body area burns in adults (>10% children)

 The main aim of resuscitation is to prevent the burn deepening  Most fluid is lost 24 hours after injury  First 8-12 hour fluid shifts are from intravascular to interstitial fluid compartments  Therefore circulatory volume can be compromised. However fluid resuscitation causes more fluid into the interstitial compartment especially colloid (therefore avoided in first 8-24 hours)  Protein loss occurs

Fluid resuscitation formula

Parkland formula (Crystalloid only e.g. Hartman's solution/Ringers' lactate)

Total fluid requirement in 24 hours = 4 ml x (total burn surface area (%)) x (body weight (kg))

 50% given in first 8 hours  50% given in next 16 hours

Resuscitation endpoint:Urine output of 0.5-1.0 ml/kg/hour in adults (increase rate of fluid to achieve this)

Points to note:

 Starting point of resuscitation is time of injury  Deduct fluids already given

After 24 hours

 Maintenance crystalloid (usually dextrose-saline) is continued at a rate of 1.5 ml x(burn area)x(body weight)  Colloids are rarely used (e.g. albumin)  Antioxidants, such as vitamin C, can be used to minimize oxidant-mediated contributions to the inflammatory cascade in burns  High tension electrical injuries and inhalation injuries require more fluid  Monitor: packed cell volume, plasma sodium, base excess, and lactate

Next question

A 23 year old man sustains a severe facial fracture and reconstruction is planned. Which of the following investigations will facilitate pre-operative planning?

Mandibular tomography

Magnetic resonance scan of face

Skull X-ray

Computerised tomography of the head

Orthopantomogram

Theme from 2011 Exam Significant facial fractures may have intracranial communication. CT scanning will allow delineation of injury extent and 3D reconstruction images can be created. An Orthopantomogram (OPT) will provide good images of mandible and surrounding bony structures but will not give intracranial detail. A skull x-ray lacks the detail for modern practice. Please rate this question:

Discuss and give feedback Next question Craniomaxillofacial injuries

Craniomaxillofacial injuries in the UK are due to:

 Interpersonal violence (52%)  Motor vehicle accidents (16%)  Sporting injuries (19%)  Falls (11%)

Le Fort Fractures Grade Feature

Le The fracture extends from the nasal septum to the lateral pyriform rims, travels horizontally above the teeth apices, crosses below the zygomaticomaxillary junction, and Grade Feature

Fort 1 traverses the pterygomaxillary junction to interrupt the pterygoid plates.

Le These fractures have a pyramidal shape and extend from the nasal bridge at or below the Fort 2 nasofrontal suture through the frontal process of the maxilla, inferolaterally through the lacrimal bones and inferior orbital floor and rim through or near the inferior orbital foramen, and inferiorly through the anterior wall of the maxillary sinus; it then travels under the zygoma, across the pterygomaxillary fissure, and through the pterygoid plates.

Le These fractures start at the nasofrontal and frontomaxillary sutures and extend posteriorly Fort 3 along the medial wall of the orbit through the nasolacrimal groove and ethmoid bones. The thicker sphenoid bone posteriorly usually prevents continuation of the fracture into the optic canal. Instead, the fracture continues along the floor of the orbit along the inferior orbital fissure and continues superolaterally through the lateral orbital wall, through the zygomaticofrontal junction and the zygomatic arch. Intranasally, a branch of the fracture extends through the base of the perpendicular plate of the ethmoid, through the vomer, and through the interface of the pterygoid plates to the base of the sphenoid. This type of fracture predisposes the patient to CSF rhinorrhea more commonly than the other types.

Ocular injuries Superior orbital fissure syndrome Severe force to the lateral wall of the orbit resulting in compression of neurovascular structures. Results in :

 Complete opthalmoplegia and ptosis (Cranial nerves 3, 4, 6 and nerve to levator palpebrae superioris)  Relative afferent pupillary defect  Dilatation of the pupil and loss of accommodation and corneal reflexes  Altered sensation from forehead to vertex (frontal branch of trigeminal nerve)

Orbital blow out fracture Typically occurs when an object of slightly larger diameter than the orbital rim strikes the incompressible eyeball. The bone fragment is displaced downwards into the antral cavity, remaining attached to the orbital periosteum. Periorbital fat may be herniated through the defect, interfering with the inferior rectus and inferior oblique muscles which are contained within the same fascial sheath. This prevents upward movement and outward rotation of the eye and the patient experiences diplopia on upward gaze. The initial bruising and swelling may make assessment difficult and patients should usually be reviewed 5 days later. Residual defects may require orbital floor reconstruction.

Nasal Fractures

 Common injury  Ensure new and not old deformity  Control epistaxis  CSF rhinorrhoea implies that the cribriform plate has been breached and antibiotics will be required.  Usually best to allow bruising and swelling to settle and then review patient clinically. Major persistent deformity requires fracture manipulation, best performed within 10 days of injury.

Retrobulbar haemorrhage Rare but important ocular emergency. Presents with:

 Pain (usually sharp and within the globe)  Proptosis  Pupil reactions are lost  Paralysis (eye movements lost)  Visual acuity is lost (colour vision is lost first)

May be the result of Le Fort type facial fractures.

Management:

 Mannitol 1g/Kg as 20% infusion, Osmotic diuretic, Contra-indicated in congestive heart failure and pulmonary oedema  Acetazolamide 500mg IV, (Monitor FBC/U+E) Reduces aqueous pressure by inhibition of carbonic anhydrase (used in glaucoma)  Dexamethasone 8mg orally or intravenously  In a traumatic setting an urgent cantholysis may be needed prior to definitive surgery.

Consider Papaverine 40mg smooth muscle relaxant Dextran 40 500mls IV improves perfusion Next question

A 23 year old man is stabbed in the right upper quadrant and is haemodynamically unstable. A laparotomy is performed and the liver has some extensive superficial lacerations and is bleeding profusely. The patient becomes progressively more haemodynamically unstable. What is the best management option?

Pack the liver and close the abdomen

Occlude the hepatic inflow with a pringles manoeuvre and suture the defects

Occlude vascular inflow and resect the most severely affected area anatomically

Perform a portosystemic shunt procedure

Suture the defects without vascular occlusion

Packing of the liver is the safest option and resection or repair considered later when the physiology is normalised. Often when the packs are removed all the bleeding has ceased and the abdomen can be closed without further action. Definitive attempts at suturing or resection at the primary laparotomy are often complicated by severe bleeding. Please rate this question:

Discuss and give feedback Next question Trauma management

The cornerstone of trauma management is embodied in the principles of ATLS.

Following trauma there is a trimodal death distribution:

 Immediately following injury. Typically as result of brain or high spinal injuries, cardiac or great vessel damage. Salvage rate is low.  In early hours following injury. In this group deaths are due to phenomena such as splenic rupture, sub dural haematomas and haemopneumothoraces  In the days following injury. Usually due to sepsis or multi organ failure.

Aspects of trauma management

 ABCDE approach.  Tension pneumothoraces will deteriorate with vigorous ventilation attempts.  External haemorrhage is managed as part of the primary survey. As a rule tourniquets should not be used. Blind application of clamps will tend to damage surrounding structures and packing is the preferred method of haemorrhage control.  Urinary catheters and naso gastric tubes may need inserting. Be wary of basal skull fractures and urethral injuries.  Patients with head and neck trauma should be assumed to have a cervical spine injury until proven otherwise.

Thoracic injuries

 Simple pneumothorax  Mediastinal traversing wounds  Tracheobronchial tree injury  Haemothorax  Blunt cardiac injury  Diaphragmatic injury  Aortic disruption  Pulmonary contusion

Management of thoracic trauma

 Simple pneumothorax insert chest drain. Aspiration is risky in trauma as pneumothorax may be from lung laceration and convert to tension pneumothorax.  Mediastinal traversing wounds These result from situations like stabbings. Exit and entry wounds in separate hemithoraces. The presence of a mediastinal haematoma indicates the likelihood of a great vessel injury. All patients should undergo CT angiogram and oesophageal contrast swallow. Indications for thoracotomy are largely related to blood loss and will be addressed below.  Tracheobronchial tree injury Unusual injuries. In blunt trauma most injuries occur within 4cm of the carina. Features suggesting this injury include haemoptysis and surgical emphysema. These injuries have a very large air leak and may have tension pneumothorax.  Haemothorax Usually caused by laceration of lung vessel or internal mammary artery by rib fracture. Patients should all have a wide bore 36F chest drain. Indications for thoracotomy include loss of more than 1.5L blood initially or ongoing losses of >200ml per hour for >2 hours.  Cardiac contusions Usually cardiac arrhythmias, often overlying sternal fracture. Perform echocardiography to exclude pericardial effusions and tamponade. Risk of arrhythmias falls after 24 hours.  Diaphragmatic injury Usually left sided. Direct surgical repair is performed.  Traumatic aortic disruption Commonest cause of death after RTA or falls. Usually incomplete laceration near ligamentum arteriosum. All survivors will have contained haematoma. Only 1-2% of patients with this injury will have a normal chest x-ray.  Pulmonary contusion Common and lethal. Insidious onset. Early intubation and ventilation.

Abdominal trauma

 Deceleration injuries are common.  In blunt trauma requiring laparotomy the spleen is most commonly injured (40%)  Stab wounds traverse structures most commonly liver (40%)  Gunshot wounds have variable effects depending upon bullet type. Small bowel is most commonly injured (50%)  Patients with stab wounds and no peritoneal signs up to 25% will not enter the peritoneal cavity  Blood at urethral meatus suggests a urethral tear  High riding prostate on PR = urethral disruption  Mechanical testing for pelvic stability should only be performed once

Investigations in

Diagnostic Peritoneal Abdominal CT scan USS Lavage

Indication Document bleeding if Document organ injury if Document fluid if hypotensive normotensive hypotensive

Advantages Early diagnosis and Most specific for Early diagnosis, non sensitive; 98% accurate localising injury; 92 to invasive and repeatable; 98% accurate 86 to 95% accurate

Disadvantages Invasive and may miss Location of scanner away Operator dependent and retroperitoneal and from facilities, time taken may miss diaphragmatic injury for reporting, need for retroperitoneal injury contrast

 Amylase may be normal following pancreatic trauma  Urethrography if suspected urethral injury

Next question

A 62 year old woman presents with acute bowel obstruction. She has been vomiting up to 15 times a day and is taking erythromycin. She suddenly complains of dizziness. Her ECG shows torsades de pointes. What is the management of choice?

IV Atropine

IV Potassium

IV Magnesium sulphate

IV Bicarbonate

IV Adrenaline

Torsades de pointes: Treatment IV magnesium sulphate

This woman is likely to have hypokalaemia and hypomagnasaemia as a result of vomiting. In addition to this, the erythromycin will predispose her to torsades de pointes. The patient needs Magnesium 2g over 10 minutes. Knowledge of the management of this peri arrest diagnosis is hence important in surgical practice. Please rate this question:

Discuss and give feedback Next question Torsades de pointes

Torsades de pointes ('twisting of the points') is a rare arrhythmia associated with a long QT interval. It may deteriorate into ventricular fibrillation and hence lead to sudden death

Causes of long QT interval

 congenital: Jervell-Lange-Nielsen syndrome, Romano-Ward syndrome  antiarrhythmics: amiodarone, sotalol, class 1a antiarrhythmic drugs  tricyclic antidepressants  antipsychotics  chloroquine  terfenadine  erythromycin  electrolyte: hypocalcaemia, hypokalaemia, hypomagnesaemia  myocarditis  hypothermia  subarachnoid haemorrhage

Management

 IV magnesium sulphate

Next question

A 27 year old man sustains a single to the left thigh. In the emergency department, he is noted to have a large haematoma of his medial thigh. He complains of parasthesia in his foot. On examination, there are weak pulses palpable distal to the injury and the patient is unable to move his foot. The appropriate initial management of this patient is:

Conventional angiography

Immediate exploration and repair

Fasciotomy of the anterior compartment

Observation for resolution of spasm

Local wound exploration

The five P's of arterial injury include pain, parasthesias, pallor, pulselessness and paralysis. In the extremities, the tissues most sensitive to anoxia are the peripheral nerves and striated muscle. The early developments of paresthesias and paralysis are signals that there is significant ischemia present, and immediate exploration and repair are warranted. The presence of palpable pulse does not exclude an arterial injury because this presence may represent a transmitted pulsation through a blood clot. When severe ischemia is present, the repair must be completed within 6 to 8 h to prevent irreversible muscle ischemia and loss of limb function. Delay to obtain a conventional angiogram or to observe for change needlessly prolongs the ischemic time. A CT angiogram may be a reasonable alternative. Fasciotomy may be required but should be done in conjunction with and after re- establishment of arterial flow. Local wound exploration is not recommended because brisk hemorrhage may be encountered without the securing of prior vascular control. Please rate this question:

Discuss and give feedback Next question Vascular trauma

Peripheral and central vessels may be injured by blunt, penetrating or shearing injuries. Fractures of bones close to vessels may also be associated with vascular injury or vessel occlusion.

Assessment

 Check for signs of distal perfusion  Doppler signal distally (monophasic/ biphasic or triphasic)  Anatomical location (which vessel is likely to be involved)  Duplex scanning and angiography are "gold standard" tests but may not be immediately available in the trauma setting

Management

 Almost always operative.  Obtaining proximal and distal control of affected vessels is crucial.  Simple lacerations of arteries may be directly closed, or a vein patch applied if there is a risk of subsequent stenosis.  Transection of the vessel should be treated by either end to end anastomosis (often not possible) or an interposition vein graft.  Use of PTFE in traumatic open injuries will invariably result in infection.

Next question

Theme: Paediatric emergencies

A. Manage conservatively B. Immediate emergency theatre C. Treat in emergency department D. Treat in emergency department under sedation E. Operate on next emergency list

Please select the most appropriate intervention for the scenario given. Each option may be used once, more than once or not at all.

21. A 3 year old child inserts a crayon into their external auditory meatus. Attempts to remove it have not been successful.

You answered Manage conservatively

The correct answer is Operate on next emergency list

Theme from September 2011 Exam They would not tolerate removal in the emergency department. The tympanic membrane should be carefully inspected and again this will be easier under general anaesthesia.

22. A 2 year old accidentally inhales a peanut. They arrive in the emergency department extremely distressed and cyanotic. Imaging shows it to be lodged in the left main bronchus.

You answered Manage conservatively

The correct answer is Immediate emergency theatre

As they are cyanosed it requires immediate removal and this should be undertaken in a fully staffed theatre. Ideally a rigid bronchoscopy should be performed.

23. A 10 year old boy is shot in the head with an airgun pellet. He is concerned that he will get into trouble and the injury remains concealed for 10 days. Imaging using CT scanning shows it to be lodged in the frontal lobe.

Manage conservatively

The pellet is small and no serious injury has occurred at this stage. This should therefore be managed conservatively.

Please rate this question:

Discuss and give feedback Next question Management of acute cases- Paediatric

 Children will often insert objects into orifices such as the nose and external auditory meatus  Assessment includes assessment of airway and haemodynamic status  Where the airway is not immediately threatened decisions can be made as to whether to manage in the emergency department or transfer to theatre  In general children do not tolerate procedures well and it is usually safer to remove objects in theatre and under general anaesthesia with a secure airway  A chest x-ray is required to ensure that no object is present in the chest, not all objects are radioopaque. However, signs such as focal consolidation may indicate small airway obstruction  In the case of small bore missile injuries the decision relating to surgery depends on the size of the missile and its location. Airgun pellets are a common culprit, if there is a long time interval between the incident and presentation and the object has not caused any significant problems then it may be best left alone  Airgun pellets (and glass) lodged in the soft tissues are usually notoriously difficult to localise and extract, no matter how superficial. Removal in theatre is usually the best option. If the object is radioopaque then an image intensifier should be used

Next question

A 32 year old male is receiving a blood transfusion after being involved in a road traffic accident. A few minutes after the transfusion he complains of loin pain. His observations show temperature 39 oC, HR 130bpm and blood pressure is 95/40mmHg. What is the best test to confirm his diagnosis?

USS abdomen

Direct Coomb's test

Blood cultures

Blood film

Sickle cell test

Mnemonic for transfusion reactions:

Got a bad unit

G raft vs. Host disease O verload T hrombocytopaenia

A lloimmunization

B lood pressure unstable A cute haemolytic reaction D elayed haemolytic reaction

U rticaria N eutrophilia I nfection T ransfusion associated lung injury

The diagnosis is of an acute haemolytic transfusion reaction, normally due to ABO incompatibility. Haemolysis of the transfused cells occurs causing the combination of shock, haemoglobinaemia and loin pain. This may subsequently lead to disseminated intravascular coagulation. A Coomb's test should confirm haemolysis. Other tests for haemolysis include: unconjugated bilirubin, haptoglobin, serum and urine free haemoglobin.

Note that delayed haemolytic reactions are normally associated with antibodies to the Rh system and occur 5-10 days after transfusion.

Please rate this question:

Discuss and give feedback

Next question

Blood transfusion reactions

Acute transfusion reactions present as adverse signs or symptoms during or within 24 hours of a blood transfusion. The most frequent reactions are fever, chills, pruritus, or urticaria, which typically resolve promptly without specific treatment or complications. Other signs occurring in temporal relationship with a blood transfusion, such as severe dyspnoea, pyrexia, or loss of consciousness may be the first indication of a more severe potentially fatal reaction. The causes of adverse reactions are multi-factorial. Immune mediated reactions, some of the most feared, occur as a result of component mismatch, the commonest cause of which is clerical error. More common, non immune mediated, complications may occur as a result of product contamination, this may be bacterial or viral. Transfusion related lung injury is well recognised and there are two proposed mechanisms which underpin this. One involves the sequestration of primed neutrophils within the recipient pulmonary capillary bed. The other proposed mechanism suggests that HLA mismatches between donor neutrophils and recipient lung tissue is to blame. The table below summarises the main types of transfusion reaction.

Immune mediated Non immune mediated

Pyrexia Hypocalcaemia

Alloimmunization CCF

Thrombocytopaenia Infections

Transfusion associated lung injury Hyperkalaemia Immune mediated Non immune mediated

Graft vs Host disease

Urticaria

Acute or delayed haemolysis

ABO incompatibility

Rhesus incompatibility

Next question

A 45 year old man complains of sharp chest pain. He is due to have elective surgery to replace his left hip. He has been bed bound for 3 months. He suddenly collapses; his blood pressue is 70/40mmHg, heart rate 120 bpm and his saturations are 74% on air. He is deteriorating in front of you. What is the next best management plan?

Aspirin

Thrombolysis with Alteplase

Unfractionated heparin

Thrombolysis with streptokinase

Clopidogrel

This man is peri arrest with the diagnosis of pulmonary (chest pain,bedbound, collapse, low saturations). He needs urgent thrombolysis with alteplase (he may not survive if you wait for the medical Spr/ITU to arrive!). Please rate this question:

Discuss and give feedback Next question Pulmonary embolism: management

A summary of the British Thoracic Society guidelines

 Heparin should be given if intermediate or high clinical probability before imaging.  Unfractionated heparin (UFH) should be considered (a) as a first dose bolus, (b) in massive PE, or (c) where rapid reversal of effect may be needed.  Otherwise, low molecular weight heparin (LMWH) should be considered as preferable to UFH, having equal efficacy and safety and being easier to use.  Oral anticoagulation should only be commenced once VTE has been reliably confirmed.  The target INR should be 2.0-3.0; when this is achieved, heparin can be discontinued.  The standard duration of oral anticoagulation is: 4 to 6 weeks for temporary risk factors, 3 months for first idiopathic, and at least 6 months for other; the risk of bleeding should be balanced with that of further VTE.

Massive PE

 CTPA or echocardiography will reliably diagnose clinically massive PE.  Thrombolysis is 1st line for massive PE (ie circulatory failure) and may be instituted on clinical grounds alone if cardiac arrest is imminent; a 50 mg bolus of alteplase is recommended.  Invasive approaches (thrombus fragmentation and IVC filter insertion) should be considered where facilities and expertise are readily available.

Next question

A 30 year old woman, who is 30 weeks pregnant, attends the varicose vein clinic. The patient is normally fit and well, with no lung disorders. She suddenly complains of shortness of breath and chest pain. She has no underlying lung condition. Her saturations are 92 % air, blood pressure 150/80 mmHg and her chest sounds clear. What would be the main investigation recommended to establish her diagnosis?

Lung spirometry

Half dose scintigraphy

CTPA

Echocardiogram

Full dose scintigraphy

The main differential diagnosis is pulmonary embolism. A CXR should be performed first in second and third trimester to exclude other diagnoses such as pneumothorax or pneumonia. Concerns surrounding radiation exposure by CXR have been discounted at this stage, as the need to establish a diagnosis is the major priority. If the chest x-ray is normal, then half dose scintigraphy should be performed. If there is underlying lung pathology, then a CTPA is performed.

This is a controversial topic area, however the answer is related to guidance from the Royal College of Obstetricians and Gynaecologists. The official line is that half dose scintigraphy is undertaken in those with normal CXR (or no chest signs) and no pre- existing lung disease. The reason for this is the significant subsequent increase in risk of maternal breast cancer with CTPA (lifetime risk increased by up to 13.6% with CTPA, background risk of 1/200 for study population) vs scintigraphy. The vast majority of female patients are not found to have a PE which is also an important component. However, we recognise that in most hospitals a CTPA is still performed first line. Reference The acute management of thrombosis and embolism during pregnancy and the puerperium RCOG guidelines February 2007

Scarsbrook A.Fand Gleeson V. Investigating suspected pulmonary embolism in pregnancy. BMJ 2007 (326) : 1135 doi: 10.1136/bmj.7399.1135. Please rate this question:

Discuss and give feedback Next question Chest pain in pregnancy Aortic dissection

 Predisposing factors in pregnancy are hypertension, congenital heart disease and Marfan's syndrome  Mainly Stanford type A dissections  Sudden tearing chest pain, transient syncope  Patient may be cold and clammy, hypertensive and have an aortic regurgitation murmur  Involvement of the right coronary artery may cause inferior myocardial infarction

Surgical management Gestational timeframe Management

< 28/40 Aortic repair with the fetus kept in utero

28-32/40 Dependent on fetal condition

> 32/40 Primary Cesarean section followed by aortic repair at the same operation

Mitral stenosis

 Most cases associated with rheumatic heart disease  Becoming less common in British women; suspect in Immigrant women  Commonest cardiac condition in pregnancy  Commonly associated with mortality  Valve surgery; balloon valvuloplasty preferable

Pulmonary embolism

 Leading cause of mortality in pregnancy  Half dose scintigraphy; CT chest if underlying lung disease, should aid diagnosis  Treatment with low molecular weight heparin throughout pregnancy and 4-6 weeks after childbirth  Warfarin is contra indicated in pregnancy (though may be continued in women with mechanical heart valves due to the significant risk of thromboembolism)

References 1. Bates S.M. and Ginsberg J.S. How we manage venous thromboembolism during pregnancy. Blood2002 (100): 3470-3478.

2. Scarsbrook A.Fand Gleeson V. Investigating suspected pulmonary embolism in pregnancy. BMJ2007 (326) : 1135 doi: 10.1136/bmj.7399.1135.

3. Morley C. A. and Lim B. A. Lesson of the Week: The risks of delay in diagnosis of breathlessness in pregnancy. BMJ 1995 (311) : 1083. Next question

Theme: Bleeding disorders

A. Vitamin K deficiency

B. von Willebrand's disease

C. Acquired haemophilia

D. Haemophilia B

E. Protein C deficiency

F. Disseminated intravascular coagulation

G. Factor V Leiden

H. Excess heparin

I. Warfarin overdose

J. Antiphospholipid syndrome

What is the most likely diagnosis for the scenario given? Each option may be used once, more than once or not at all.

27. A 33 year old female is admitted for varicose vein surgery. She is fit and well. After the procedure she is persistently bleeding. She is known to have menorrhagia. Investigations show a prolonged bleeding time and increased APTT. She has a normal PT and platelet count.

You answered Vitamin K deficiency

The correct answer is von Willebrand's disease

Bleeding post operatively, epistaxis and menorrhagia may indicate a diagnosis of vWD. Haemoarthroses are rare. The bleeding time is usually normal in haemophilia (X-linked) and vitamin K deficiency.

28. A 70 year old heavy smoker presents with 3 weeks of haematuria and bruising. He is normally fit and well. He is on no medications. His results reveal: Hb 9.0 WCC 11 Pl 255 PT 16 (normal) APTT 58 (increased) Thrombin time 20 (normal).

You answered Vitamin K deficiency

The correct answer is Acquired haemophilia

This patient has Factor 8 acquired disorder. He is likely to have developed a lung malignancy (smoker) and as a result aquired a haemophilia disorder. The elderly, pregnancy, malignancy and autoimmune conditions are associated with acquired haemophilia. Prolonged APTT is key to the diagnosis. Management involves steroids.

29. A 28 year old female is attends the gynaecology unit for a D+C following an incomplete miscarriage. She has previously had recurrent pulmonary embolic events. After the procedure she is persistently bleeding. Her APTT is 52 (increased).

You answered Vitamin K deficiency

The correct answer is Antiphospholipid syndrome

A combination of thromboembolism and bleeding in a young woman should raise the possibility of antiphospholipid syndrome. Other features may include foetal loss, venous and arterial thrombosis and thrombocytopenia. A Lupus anticoagulant may be present and the APTT is prolonged.

Theme from January 2012 exam

Please rate this question:

Discuss and give feedback

Next question Abnormal coagulation

Cause Factors affected

Heparin Prevents activation factors 2,9,10,11

Warfarin Affects synthesis of factors 2,7,9,10

DIC Factors 1,2,5,8,11

Liver disease Factors 1,2,5,7,9,10,11

Interpretation blood clotting test results

Disorder APTT PT Bleeding time

Haemophilia Increased Normal Normal von Willebrand's disease Increased Normal Increased

Vitamin K deficiency Increased Increased Normal

Next question

A 54-year-old man is brought to the Emergency Department after being found collapsed in the street. He is known to have a history of alcoholic liver disease. Blood tests reveal the following:

Calcium 1.62 mmol/l

Albumin 33 g/l

Which one of the following is the most appropriate management of the calcium result?

10ml of 10% calcium chloride over 10 minutes

20% albumin infusion

10ml of 50% calcium gluconate over 10 minutes

No action

10ml of 10% calcium chloride over 4 hours

Current UK ALS guidance is to use calcium chloride

Even after correction for the low albumin level this patient has significant hypocalcaemia which should be corrected. Please rate this question:

Discuss and give feedback Next question Hypocalcaemia: causes and management

The clinical history combined with parathyroid hormone levels will reveal the cause of hypocalcaemia in the majority of cases

Causes

 Vitamin D deficiency (osteomalacia)  Acute pancreatitis  Chronic renal failure  Hypoparathyroidism (e.g. post thyroid/parathyroid surgery)  Pseudohypoparathyroidism (target cells insensitive to PTH)  (initial stages)  Magnesium deficiency (due to end organ PTH resistance)

Management

 Acute management of severe hypocalcaemia is with intravenous replacement. The preferred method is with intravenous calcium chloride, 10ml of 10% solution over 10 minutes  ECG monitoring is recommended  Further management depends on the underlying cause  Calcium and bicarbonate should not be administered via the same route

Next question

A 25 year old male pedestrian is hit by a van on a busy road. He is brought to the Emergency Department by ambulance. On examination he is dyspneoic, and hypoxic despite administration of high flow 100% oxygen. His blood pressure is 110/70 and pulse rate is 115 bpm. The right side of his chest is hyper-resonant on percussion and has decreased breath sounds. The trachea is deviated to the left. What is the most likely underlying diagnosis?

Fat embolism

Tension pneumothorax

Rupture of the right main bronchus

Rupture of the diaphragm

Pulmonary contusion

Blunt or penetrating chest trauma that creates a flap type defect on the surface of the lung can result in a tension pneumothorax. Typical features include dyspnoea, progressive hypoxia, hyperresonance and tracheal deviation. Treatment is with needle decompression and chest tube insertion. Please rate this question:

Discuss and give feedback Next question Thoracic trauma

Types of thoracic trauma

Tension  Often laceration to lung parenchyma with flap pneumothorax  Pressure develops in thorax  Most common cause is mechanical ventilation in patient with pleural injury  Symptoms overlap with cardiac tamponade, hyper-resonant percussion note is more likely in tension pnemothorax

Flail chest  Chest wall disconnects from thoracic cage  Multiple rib fractures (at least two fractures per rib in at least two ribs)  Associated with pulmonary contusion  Abnormal chest motion  Avoid over hydration and fluid overload

Pneumothorax  Most common cause is lung laceration with air leakage  Most traumatic pneumothoraces should have a chest drain  Patients with traumatic pneumothorax should never be mechanically ventilated until a chest drain is inserted

Haemothorax  Most commonly due to laceration of lung, intercostal vessel or internal mammary artery  Haemothoraces large enough to appear on CXR are treated with large bore chest drain  Surgical exploration is warranted if >1500ml blood drained immediately

Cardiac tamponade  Beck's triad: elevated venous pressure, reduced arterial pressure, reduced heart sounds  Pulsus paradoxus  May occur with as little as 100ml blood

Pulmonary contusion  Most common potentially lethal chest injury  Arterial blood gases and pulse oximetry important  Early intubation within an hour if significant hypoxia

Blunt cardiac injury  Usually occurs secondary to chest wall injury  ECG may show features of myocardial infarction  Sequelae: hypotension, arrhythmias, cardiac wall motion abnormalities

Aorta disruption  Deceleration injuries  Contained haematoma  Widened mediastinum

Diaphragm  Most due to motor vehicle accidents and blunt trauma causing disruption large radial tears (laceration injuries result in small tears)  More common on left side  Insert gastric tube, may pass into intrathoracic stomach Mediastinal  Entrance wound in one hemithorax and exit wound/foreign body traversing wounds in opposite hemithorax  Mediastinal haematoma or pleural cap suggests great vessel injury  Mortality is 20%

References ATLS Manual 8th Edition Next question

Theme: Management of skin injuries

A. Wound excision and primary closure B. Simple primary closure C. Delayed primary closure D. Debridement and healing by secondary intention E. Split thickness skin graft F. Full thickness skin graft G. Free flap H. Pedicled flap I. Debridement and rotational flap

For the injuries described please select the most appropriate management. Each option may be used once, more than once or not at all.

32. A 32 year old man is involved in a road traffic accident and sustains a significant laceration to the lateral aspect of the nose which is associated with tissue loss.

You answered Wound excision and primary closure

The correct answer is Debridement and rotational flap

Theme from April 2011 Exam Nasal injuries can be challenging to manage and where there is tissue loss, it can be difficult to primarily close them and still obtain a satisfactory aesthetic result. Debridement together with a rotational flap would obtain the best results here.

33. A 7 year old boy falls over and sustains a 6cm laceration to his head. On inspection his wound contains some dirt in it.

Wound excision and primary closure

By debriding the wound, the area can then be primarily closed. Prophylactic antibiotics should be administered.

34. A 45 year old man is gardening and damages his foot with a fork. On examination there are cutaneous defects and the surrounding skin looks dusky.

You answered Wound excision and primary closure

The correct answer is Debridement and healing by secondary intention

The skin changes described here should be debrided. Closure would not be safe with the skin changes documented and the wound should be left open.

Please rate this question:

Discuss and give feedback Next question Methods of wound closure

Method of Indication closure

Primary closure  Clean wound, usually surgically created or following minor trauma  Standard suturing methods will usually suffice  Wound heals by primary intention

Delayed primary  Similar methods of actual closure to primary closure closure  May be used in situations where primary closure is either not achievable or not advisable e.g. infection

Vacuum assisted  Uses negative pressure therapy to facilitate wound closure closure  Sponge is inserted into wound cavity and then negative pressure applied  Advantages include removal of exudate and versatility  Disadvantages include cost and risk of fistulation if used incorrectly on sites such as bowel

Split thickness  Superficial dermis removed with Watson knife or dermatome skin grafts (commonly from thigh)  Remaining epithelium regenerates from dermal appendages  Coverage may be increased by meshing

Full thickness skin  Whole dermal thickness is removed grafts  Sub dermal fat is then removed and graft placed over donor site  Better cosmesis and flexibility at recipient site  Donor site "cost"

Flaps  Viable tissue with a blood supply  May be pedicled or free  Pedicled flaps are more reliable, but limited in range  Free flaps have greater range but carry greater risk of breakdown as they require vascular anastomosis

Next question

A 19 year old student is involved in a head on car collision. He complains of severe chest pain. A Chest x-ray performed as part of a trauma series shows widening of the mediastinum. Which is the most likely injury in this scenario?

Rupture of the distal oesophagus

Rupture of the left main bronchus

Rupture of the aorta proximal to the left subclavian artery

Rupture of the aorta distal to the left subclavian artery

Rupture of the inferior vena cava

The aorta may be injured in deceleration accidents. In the setting of deceleration injury, chest pain and mediastinal widening the most likely problem is aortic rupture. This will typically occur distal to the left subclavian artery. Rupture of the proximal aorta may occur. However, survival is unlikely. It is important to note that the question uses the term Most likely injury as this is the component that distinguishes an ascending rupture from a descending rupture. Please rate this question:

Discuss and give feedback Next question Thoracic aorta rupture

 Mechanism of injury: Decelerating force i.e. RTA, fall from a great height  Most people die at scene  Survivors may have an incomplete laceration at the ligamentum arteriosum of the aorta.

Clinical features

 Contained haematoma: persistent hypotension  Detected mainly by history, CXR changes

CXR changes

 Widened mediastinum  Trachea/Oesophagus to right  Depression of left main stem bronchus  Widened paratracheal stripe/paraspinal interfaces  Space between aorta and pulmonary artery obliterated  Rib fracture/left haemothorax

Diagnosis Angiography, usually CT aortogram.

Treatment Repair or replacement. Ideally they should undergo endovascular repair. Next question

Theme: Management of head and neck trauma

A. Observation B. CT head within 1 hour C. CT head within 8 hours D. Urgent neurosurgical review (even before CT head performed) E. 3 view c-spine xray F. 2 view c-spine xray G. CT c-spine

What is the best initial management plan for the injuries described? Each option may be used once, more than once or not at all.

36. A 22 year old mechanic is involved in a fight. He is hit on the head with a hammer. On examination he had clinical evidence of an open depressed skull fracture and a GCS of 6/15.

You answered Observation

The correct answer is Urgent neurosurgical review (even before CT head performed)

A patient with GCS <8 or = to 8 needs urgent neurosurgical review. Especially when an open fracture is present.

37. A 67 year old retired lawyer falls down the stairs. His GCS is 15/15 and he has some bruising over the mastoid.

You answered Observation

The correct answer is CT head within 1 hour

This patient has a basal skull fracture, which is indicated by a positive Battle's sign. He should have a CT head within 1h.

38. A 52 year old secretary falls down the stairs. She complains of neck pain. She has a GCS of 15/15 and no neurology. She is unable to rotate her c-spine 45 degrees to the left and right.

You answered Observation

The correct answer is 3 view c-spine xray

In the January 2014 NICE guidance relating to the diagnosis of head and spinal injury the evidence for initial imaging of the C spine was reviewed. The current UK practice is that "low risk" patients with pain but no neurology undergo a 3 view C spine x-ray. The more detailed 5 view x-ray was not found to be any better than 3 view films. In patients with high risk injuries (which the patient in the scenario does not have) there is a likelihood that 1 in 6 injuries would be missed on plain films alone and therefore CT scanning of the C spine is recommended in this group.

Please rate this question:

Discuss and give feedback Next question Head injury management- NICE Guidelines

Summary of guidelines

 All patients should be assessed within 15 minutes on arrival to A&E  Document all 3 components of the GCS  If GCS <8 or = to 8, consider stabilising the airway  Treat pain with low dose IV opiates (if safe)  Full spine immobilisation until assessment if:

- GCS < 15 - neck pain/tenderness - paraesthesia extremities - focal neurological deficit - suspected c-spine injury

If a c-spine injury is suspected a 3 view c-spine x-ray is indicated. CT c-spine is preferred if: - Intubated - GCS <13 - Normal x-ray but continued concerns regarding c-spine injury - Any focal neurology - A CT head scan is being performed - Initial plain films are abnormal

Immediate CT head (within 1 hour) if:

 GCS < 13 on admission  GCS < 15 2 hours after admission  Suspected open or depressed skull fracture  Suspected skull base fracture (panda eyes, Battle's sign, CSF from nose/ear, bleeding ear)  Focal neurology  Vomiting > 1 episode  Post traumatic seizure  Coagulopathy

Contact neurosurgeon if:

 Persistent GCS < 8 or = 8  Unexplained confusion > 4h  Reduced GCS after admission  Progressive neurological signs  Incomplete recovery post seizure  Penetrating injury  Cerebrospinal fluid leak

Observations

 1/2 hourly GCS until 15

Reference 1. http://guidance.nice.org.uk/CG56/QuickRefGuide/pdf/English 2. Hodgkinson S et al. Early management of head injury: summary of NICE guidance. BMJ 2014 (348):34-37. Next question

A 28 year old African man is admitted with acute severe abdominal pain. He has just flown into the UK long haul and the pain developed whilst in flight. On examination he is tender in the left upper quadrant. His blood tests are as shown.

Hb 6 g/dl Reticulocyte count 15%. Ultrasound shows a spleen with a heterogeous texture and a few small gallstones but is otherwise normal. What is the most likely diagnosis?

Pancreatitis

Parvovirus infection

Sickle cell anaemia

Pulmonary embolism

Beta Thalassaemia minor

A combination of a high reticulocyte count and severe anaemia indicates sickle cell anaemia, however another differential can be of a transient aplastic crisis due to parvovirus. This is less likely as this causes a reticulocytopenia rather than a reticulocytosis.

Parvovirus B19 infects erythroid progenitor cells in the bone marrow and causes temporary cessation of red blood cell production, patients who have underlying hematologic abnormalities are at risk of cessation of red blood cell production if they become infected. This can result in a transient aplastic crisis. Thus, patients with sickle cell anaemia are at risk. Typically, these patients have a viral prodrome followed by anaemia, often with haemoglobin concentrations falling below 5.0 g/dL and reticulocytosis. Please rate this question:

Discuss and give feedback Next question Sickle cell anaemia

 Autosomal recessive  Single base mutation  Deoxygenated cells become sickle in shape  Causes: short red cell survival, obstruction of microvessels and infarction  Sickling is precipitated by: dehydration, infection, hypoxia  Manifest at 6 months age  Africans, Middle East, Indian  Diagnosis: Hb electrophoresis

Sickle crises

 Bone pain  Pleuritic chest pain: acute sickle chest syndrome commonest cause of death  CVA, seizures  Papillary necrosis  Splenic infarcts  Priapism  Hepatic pain

Hb does not fall during a crisis, unless there is

 Aplasia: parvovirus  Acute sequestration  Haemolysis

Long-term complications

 Infections: Streptococcus pnemoniae  Chronic leg ulcers  Gallstones: haemolysis  Aseptic necrosis of bone  Chronic renal disease  Retinal detachment, proliferative retinopathy

Surgical complications

 Bowel ischaemia  Cholecystitis  Avascular necrosis

Management

 Supportive  Hydroxyurea  Repeated transfusions pre operatively  Exchange transfusion in emergencies

Sickle cell trait

 Heterozygous state  Asymptomatic  Symptoms associated with extreme situations ie anaesthesia complications  Protective against Plasmodium falciparum

Next question

Theme: Blood transfusion reactions

A. Neutrophilic febrile reaction

B. Acute haemolytic transfusion reaction

C. Delayed haemolytic transfusion reaction

D. Pulmonary oedema

E. Sickle cell crisis

F. Transfusion associated lung injury

G. Graft vs. Host disease

Please select the most likely underlying cause for each scenario. Each option may be used once, more than once or not at all.

40. A 22 year old man is having a blood transfusion after losing blood from a peptic ulcer. He is normally fit and well. Four hours after the transfusion; he complains of sudden onset shortness of breath and chest pain. On examination his temperature is 37.2, saturations are 88% on air, blood pressure 100/55 mmHg and HR 110 bpm. He has crepitations bilaterally up to the midzones. He is given IV frusemide, but deteriorates and is admitted to ITU. A pulmonary catheter is inserted and the PCWP is 10 mmHg.

You answered Neutrophilic febrile reaction

The correct answer is Transfusion associated lung injury

The pulmonary catheter reading indicates that this is not a case of fluid overload (the PCWP should be high, normal values PCWP systolic 7mmHg, diastolic 10mmHg). Transfusion associated lung injury is a rare reaction causing neutrophilic mediated allergic pulmonary oedema. Patient's have antibodies to donor leukocytes. It is important to consider this as a diagnosis when patients don't respond to treatment for pulmonary oedema. Patients normally respond to supportive therapy including fluids and oxygen.

41. A 32 year male with leukaemia attends the day unit for a blood transfusion. Five days after the transfusion he attends A&E with a temperature of 38.5, erythroderma and desquamation.

You answered Neutrophilic febrile reaction

The correct answer is Graft vs. Host disease

This is associated with transfusion of unirradiated blood in immunosupressed patients. Transfusion associated GVHD can occur 4-30 days after a transfusion and follows a sub acute pathway. Patients may also have diarrhoea and abnormal liver function tests. Management involves steroid therapy.

42. A 22 year old man is having a blood transfusion after losing blood from haemorrhoids. He is normally fit and well. 3h during the transfusion he complains of sudden onset abdominal pain and nausea. His temperature is 39 degrees, Blood pressure 98/42 mmHg, HR 105 bpm and saturations 94% air. His urine appears dark.

You answered Neutrophilic febrile reaction

The correct answer is Acute haemolytic transfusion reaction

Rapid intravascular haemolysis leading to shock, DIC and death can occur with this reaction.

Please rate this question:

Discuss and give feedback

Next question

Blood transfusion reactions

Acute transfusion reactions present as adverse signs or symptoms during or within 24 hours of a blood transfusion. The most frequent reactions are fever, chills, pruritus, or urticaria, which typically resolve promptly without specific treatment or complications. Other signs occurring in temporal relationship with a blood transfusion, such as severe dyspnoea, pyrexia, or loss of consciousness may be the first indication of a more severe potentially fatal reaction. The causes of adverse reactions are multi-factorial. Immune mediated reactions, some of the most feared, occur as a result of component mismatch, the commonest cause of which is clerical error. More common, non immune mediated, complications may occur as a result of product contamination, this may be bacterial or viral. Transfusion related lung injury is well recognised and there are two proposed mechanisms which underpin this. One involves the sequestration of primed neutrophils within the recipient pulmonary capillary bed. The other proposed mechanism suggests that HLA mismatches between donor neutrophils and recipient lung tissue is to blame. The table below summarises the main types of transfusion reaction.

Immune mediated Non immune mediated

Pyrexia Hypocalcaemia

Alloimmunization CCF

Thrombocytopaenia Infections

Transfusion associated lung injury Hyperkalaemia

Graft vs Host disease

Urticaria

Acute or delayed haemolysis

ABO incompatibility

Rhesus incompatibility

Next question

A 49-year-old male sustained a severe blunt injury just below the bridge of the nose with industrial machinery. Imaging demonstrates a fracture involving the superior orbital fissure. On examination an ipsilateral pupillary defect is present and loss of the corneal reflexes. In addition to these examination findings, which of the following will not be present?

Altered cutaneous sensation from the forehead to the vertex

Ptosis

Complete opthalmoplegia

Nystagmus

Enopthalmos

Orbital apex syndrome This is an extension of superior orbital fissure syndrome and includes compression of the optic nerve passing through the optic foramen. It is indicated by features of superior orbital fissure syndrome and ipsilateral afferent pupillary defect.

This type of injury will result in the orbital apex syndrome (See above). As such opthalmoplegia will be present and nystagmus cannot occur. Please rate this question:

Discuss and give feedback Next question Craniomaxillofacial injuries

Craniomaxillofacial injuries in the UK are due to:

 Interpersonal violence (52%)  Motor vehicle accidents (16%)  Sporting injuries (19%)  Falls (11%)

Le Fort Fractures Grade Feature Grade Feature

Le The fracture extends from the nasal septum to the lateral pyriform rims, travels Fort 1 horizontally above the teeth apices, crosses below the zygomaticomaxillary junction, and traverses the pterygomaxillary junction to interrupt the pterygoid plates.

Le These fractures have a pyramidal shape and extend from the nasal bridge at or below the Fort 2 nasofrontal suture through the frontal process of the maxilla, inferolaterally through the lacrimal bones and inferior orbital floor and rim through or near the inferior orbital foramen, and inferiorly through the anterior wall of the maxillary sinus; it then travels under the zygoma, across the pterygomaxillary fissure, and through the pterygoid plates.

Le These fractures start at the nasofrontal and frontomaxillary sutures and extend posteriorly Fort 3 along the medial wall of the orbit through the nasolacrimal groove and ethmoid bones. The thicker sphenoid bone posteriorly usually prevents continuation of the fracture into the optic canal. Instead, the fracture continues along the floor of the orbit along the inferior orbital fissure and continues superolaterally through the lateral orbital wall, through the zygomaticofrontal junction and the zygomatic arch. Intranasally, a branch of the fracture extends through the base of the perpendicular plate of the ethmoid, through the vomer, and through the interface of the pterygoid plates to the base of the sphenoid. This type of fracture predisposes the patient to CSF rhinorrhea more commonly than the other types.

Ocular injuries Superior orbital fissure syndrome Severe force to the lateral wall of the orbit resulting in compression of neurovascular structures. Results in :

 Complete opthalmoplegia and ptosis (Cranial nerves 3, 4, 6 and nerve to levator palpebrae superioris)  Relative afferent pupillary defect  Dilatation of the pupil and loss of accommodation and corneal reflexes  Altered sensation from forehead to vertex (frontal branch of trigeminal nerve)

Orbital blow out fracture Typically occurs when an object of slightly larger diameter than the orbital rim strikes the incompressible eyeball. The bone fragment is displaced downwards into the antral cavity, remaining attached to the orbital periosteum. Periorbital fat may be herniated through the defect, interfering with the inferior rectus and inferior oblique muscles which are contained within the same fascial sheath. This prevents upward movement and outward rotation of the eye and the patient experiences diplopia on upward gaze. The initial bruising and swelling may make assessment difficult and patients should usually be reviewed 5 days later. Residual defects may require orbital floor reconstruction.

Nasal Fractures

 Common injury  Ensure new and not old deformity  Control epistaxis  CSF rhinorrhoea implies that the cribriform plate has been breached and antibiotics will be required.  Usually best to allow bruising and swelling to settle and then review patient clinically. Major persistent deformity requires fracture manipulation, best performed within 10 days of injury.

Retrobulbar haemorrhage Rare but important ocular emergency. Presents with:

 Pain (usually sharp and within the globe)  Proptosis  Pupil reactions are lost  Paralysis (eye movements lost)  Visual acuity is lost (colour vision is lost first)

May be the result of Le Fort type facial fractures.

Management:

 Mannitol 1g/Kg as 20% infusion, Osmotic diuretic, Contra-indicated in congestive heart failure and pulmonary oedema  Acetazolamide 500mg IV, (Monitor FBC/U+E) Reduces aqueous pressure by inhibition of carbonic anhydrase (used in glaucoma)  Dexamethasone 8mg orally or intravenously  In a traumatic setting an urgent cantholysis may be needed prior to definitive surgery.

Consider Papaverine 40mg smooth muscle relaxant Dextran 40 500mls IV improves perfusion Next question

Which option is not recommended during the management of compartment syndrome?

Anticoagulation

Keep limb level with the body

Intravenous fluids

Pain control

Fasciotomy

Anticoagulation will worsen compartment syndrome. Please rate this question:

Discuss and give feedback Next question Compartment syndrome

 This is a particular complication that may occur following fractures (or following ischaemia re- perfusion injury in vascular patients). It is characterised by raised pressure within a closed anatomical space.  The raised pressure within the compartment will eventually compromise tissue perfusion resulting in necrosis. The two main fractures carrying this complication include supracondylar fractures and tibial shaft injuries.

Symptoms and signs

 Pain, especially on movement (even passive)  Parasthesiae  Pallor may be present  Arterial pulsation may still be felt as the necrosis occurs as a result of microvascular compromise  Paralysis of the muscle group may occur

Diagnosis

 Is made by measurement of intracompartmental pressure measurements. Pressures in excess of 20mmHg are abnormal and >40mmHg is diagnostic.

Treatment

 This is essentially prompt and extensive fasciotomies  In the lower limb the deep muscles may be inadequately decompressed by the inexperienced operator when smaller incisions are performed  Myoglobinuria may occur following fasciotomy and result in renal failure and for this reason these patients require aggressive IV fluids  Where muscle groups are frankly necrotic at fasciotomy they should be debrided and amputation may have to be considered  Death of muscle groups may occur within 4-6 hours

Next question

Theme: Complications of burns

A. Deep vein thrombosis B. Curlings Ulcer C. D. Type I respiratory failure E. Type II respiratory failure F. Toxic shock syndrome G. Compartment syndrome H. Rhabdomyolysis I. Disseminated intravascular coagulation

For each clinical scenario please select the most likely complication to have occurred. Each option may be used once, more than once or not at all.

45. A 10 year old child is admitted with severe 30% burns following a house fire. After wound cleaning and dressings he is admitted to critical care. 1 day following skin grafts he becomes tachycardic and hypotensive. He vomits twice and this shows evidence of haematemesis

You answered Deep vein thrombosis

The correct answer is Curlings Ulcer

Stress ulcers may occur in the duodenum of burns patients and are more common in children.

46. A 26 year old electrician suffers a full thickness high voltage burn to his leg. On routine urine analysis he has + blood. His U+E's show mild hyperkalaemia and a CK of 3000

You answered Deep vein thrombosis

The correct answer is Rhabdomyolysis

Electrical high voltage burns are associated with rhabdomyolysis. Acute tubular necrosis may occur. Aggressive IV fluids should be given

47. A 45 year old man is admitted after his clothing caught fire. He suffers a full thickness circumferential burn to his lower thigh. He complains of increasing pain in lower leg and on examination there is parasthesia and severe pain in the lower leg. Foot pulses are normal

You answered Deep vein thrombosis

The correct answer is Compartment syndrome

Circumferential burns may constrict the limb and cause a compartment syndrome to develop. Eshcarotomy is required, and compartmental decompression.

Please rate this question:

Discuss and give feedback Next question Burns

Burns may be thermal, chemical or electrical. In the former category are burns which occur as a result of heat. Chemical burns occur when the skin is exposed to an extremely caustic or alkaline substance. Electrical burns occur following exposure to electrical current. The immediate management includes removal of the burning source which usually includes irrigation of the burned area. A detailed assessment then needs to be made of the extent of the burns and a number of charts are available for recording this information. The degree of injury relates to the temperature and duration of exposure. Most domestic burns are mainly scalds in young children.

Following the burn, there is a local response with progressive tissue loss and release of inflammatory cytokines. Systemically, there are cardiovascular effects resulting from fluid loss and sequestration of fluid into the third space. There is a marked catabolic response. Immunosupression is common with large burns and bacterial translocation from the gut lumen is a recognised event. Sepsis is a common cause of death following major burns.

Types of burn

Type of burn Skin layers Skin Blanching Management affected appearance

Epidermal/Superficial Epidermis Red, moist Yes

Superficial partial Epidermis and part Pale, dry Yes Normally heals thickness of papillary dermis with no affected intervention

Deep partial thickness Epidermis, whole Mottled red No Needs surgical papillary dermis colour intervention affected (depending on site) Full thickness Whole skin layer Dry, leathery No Burns centre and subcutaneous hard wound tissue affected

Depth of burn assessment

 Bleeding on needle prick  Sensation  Appearance  Blanching to pressure

Percentage burn estimation Lund Browder chart: most accurate even in children Wallace rule of nines Palmar surface: surface area palm = 0.8% burn

>15% body surface area burns in adults needs urgent burn fluid resuscitation

Transfer to burn centre if:

 Need burn shock resuscitation  Face/hands/genitals affected  Deep partial thickness or full thickness burns  Significant electrical/chemical burns

Management The initial aim is to stop the burning process and resuscitate the patient. Intravenous fluids will be required for children with burns greater than 10% of total body surface area. Adults with burns greater than 15% of total body surface area will also require IV fluids. The fluids are calculated using the Parkland formula which is; volume of fluid= total body surface area of the burn % x weight (Kg) x4. Half of the fluid is administered in the first 8 hours. A urinary catheter should be inserted. Analgesia should be given. Complex burns, burns involving the hand perineum and face and burns >10% in adults and >5% in children should be transferred to a burns unit.

Circumferential burns affecting a limb or severe torso burns impeding respiration may require escharotomy to divide the burnt tissue.

Conservative management is appropriate for superficial burns and mixed superficial burns that will heal in 2 weeks. More complex burns may require excision and skin grafting. Excision and primary closure is not generally practised as there is a high risk of infection.

There is no evidence to support the use of anti microbial prophylaxis or topical antibiotics in burn patients.

Escharotomies

 Indicated in circumferential full thickness burns to the torso or limbs.  Careful division of the encasing band of burn tissue will potentially improve ventilation (if the burn involves the torso), or relieve compartment syndrome and oedema (where a limb is involved)

References www.euroburn.org/e107files/downloads/guidelinesburncare.pdf

Barajas-Nava LA, López-Alcalde J, Roqué i Figuls M, Solà I, Bonfill Cosp X. Antibiotic prophylaxis for preventing burn wound infection. Cochrane Database of Systematic Reviews 2013, Issue 6. Art. No.: CD008738. DOI: 10.1002/14651858.CD008738.pub2.

Hettiaratchy S & Papini R. Initial management of a major burn: assessment and resuscitation. BMJ 2004;329:101-103 Next question

A 23 year old man who plays rugby for a hobby presents with recurrent anterior dislocation of the shoulder. Which of the following abnormalities is most likely to be present to account for this?

Rotator cuff tear

Biceps tendon rupture

Bankart lesion

Axillary nerve injury

Infraspinatus tendinitis

A Bankart lesion is an injury of the anterior (inferior) glenoid labrum of the shoulder due to anterior shoulder dislocation. When this happens, a pocket at the front of the glenoid forms that allows the humeral head to dislocate into it.

Anterior dislocations are the most common. When recurrent, a Bankart lesion is the most common underlying abnormality. This is usually visualised by CT and MRI scanning and often repaired arthroscopically. Please rate this question:

Discuss and give feedback Next question Shoulder disorders

Shoulder fractures and dislocations Fractures Proximal humerus Background

 Third most common fragility fracture in the elderly.  Results from low energy fall in predominantly elderly females, or from high energy trauma in young males.  Can be associated with nerve injury (commonly axillary), and fracture-dislocation of the humeral head. Detailed neurological assessment is essential for all upper limb injuries.

Anatomy Osteology Consists of articular head, greater tuberosity, lesser tuberosity, metaphysis and diaphysis. Between the articular head and the tuberosities is the anatomical neck (previous physis). Between the tuberosities and the metaphysis is the surgical neck. The supraspinatus, infraspinatus and teres minor muscles attach to the greater tuberosity. The subscapularis muscle attaches to the lesser tuberosity.

Vascular Supply Humeral head is supplied by the anterior and posterior humeral circumflex arteries. Anatomical neck fractures are at greatest risk of osteonecrosis.

Imaging Imaging aims to both delineate the fracture pattern, and confirm/exlude the presence of an associated dislocation.

 Radiographs - True anteroposterior (AP), axillary lateral and/or scapula Y view.  CT - indicated to better define intra-articular involvement and to aid pre-operative planning. MRI is not useful for fracture imaging.

Classification Description of the fracture is often more useful than classification. Particular attention should be paid to humeral alignment, fracture displacement, and greater tuberosity position (rotator cuff will pull the GT supero-posterioly, which can cause impingement problems with malunion). - Neer Classification: Most commonly used. Describes fracture as 2,3,or 4 part depending upon the number main fragments. Also comments on the degree of displacement. Fragments: -greater tuberosity -lesser tuberosity - articular surface - shaft Displacement: >1cm or angulation >45 degrees.

Treatment The vast majority of proximal humeral fractures are minimally displaced, and therefore can be managed conservatively. This involves immobilisation in a polysling, and progressive mobilisation. Pendular exercise can commence at 14 days, and active abduction from 4-6 weeks.

Irreducible fracture dislocation is an indication for operative management. Other indications include large displacement, younger patient, head splitting (intra-articular fractures). However, the recent PROFHER trial (1) has suggested no benefit to operative intervention on patient outcome (it must be applied cautiously as majority of patients were elderly with extraarticular fractures). Options available for surgical management include:

ORIF Most commonly used. Plate and screw fixation. Can reconstruct complex fractures.

Intramedullary nail Suitable for extra-articular configuration, predominantly surgical neck +/- GT fractures. Used for un-reconstructable fractures in the older patient who has good Hemiarthroplasty glenoid quality.

Total shoulder Unconstructable fractures where high functioning shoulder is required arthroplasty (hemiarthroplasty will cause glenoid erosion)

Reverse shoulder Total shoulder arthroplasty that provides better functional outcome than arthroplasty conventional total shoulder replacement.

Scapula Background Uncommon fractures usually associated with high energy trauma. Most commonly involve scapula body or spine (50%), glenoid fossa and glenoid neck. Important to exclude associated life threatening injury.

Imaging Plain radiographs should include true anteroposterior (AP), axillary lateral and/or scapula Y view. CT scanning is useful for defining intra-articular involvement, displacement and for three dimensional reconstruction.

Classification Based on the location of the fracture (coracoid, acromion, glenoid neck, glenoid fossa, scapula body). Beware of ipsilateral glenoid neck and clavicle fracture -floating shoulder - where limb is effectively dissociated from axial skeleton.

Treatment The vast majority of scapula fractures are amenable to conservative management, consisting of sling immobilisation for two weeks followed by early rehabilitation. Floating shoulder will usually require fixation, and consideration of surgery should also be given to intra-articular and displaced/angulated glenoid fractures.

Dislocations

Types Dislocations around the shoulder joint include glenohumeral dislocation, acromioclavicular joint disruption and sternoclavicular dislocation. Only glenohumeral dislocation will be covered here.

Glenohumeral dislocation Diagnosis, classification and management are covered here.

Background Shoulder dislocation is commonly seen in A&E. It has a high recurrence rate that is as high as 80% in teenagers. Initial management requires emergent reduction to prevent lasting chondral damage.

Early assessment and management Usually a traumatic cause (multi-directional instability in frequent dislocations requires discussion with orthopaedics and is not covered here). Careful history, examination and documentation of neurovascular status of the limb, in particular the axillary nerve (regimental badge sensation). This should be re-assessed post manipulation. Early radiographs to confirm direction of dislocation.

Initial management consists of emergent closed reduction under under entanox and analgesia, but often requires conscious sedation. Arm should then be immobilised in a polysling, and XR to confirm relocation.

Imaging - True anteroposterior (AP), axillary lateral and/or scapula Y view. Reduced humeral head should lie between acromion and coracoid on lateral/scapula view.

Types Reduction Direction Features Cause Examination techniques

Anterior Most Usually traumatic - anterior Loss of shoulder Common force on arm when shoulder contour - sulcus sign. Hippocratic. >90% is abducted, eternally Humeral head can be Milch. rotated felt anteriorly. Stimson.

Kocher not advised due to complication of fracture

Posterior 50% 50% traumatic, but Shoulder locked in Gentle lateral missed in classically post seizure or internal rotation. XR traction to A&E may show lightbulb adducted arm. appearance.

Inferior Rare Associated with pectorals As for primary injury Management of and rotator cuff tears, and primary injury glenoid fracture

Rare Associated with As for primary injury Management of Superior acrominon/clavicle fracture primary injury

Associated injuries

 Bankart lesion - avulsion of the anterior glenoid labrum with an anterior shoulder dislocation (reverse Bankart if poster labrum in posterior dislocation).  Hill Sachs defect - chondral impaction on posteriosuperior humeral head from contact with gleonoid rim. Can be large enough to lock shoulder, requiring open reduction. (Reverse Hill Sachs in posterior dislocation).  Rotator cuff tear - increases with age.  Greater or lesser tuberosity fracture - increases with age.  Humeral neck fracture - shoulder fracture dislocation. More common in high energy trauma and elderly. Should be discussed with orthopaedics prior to any attempted reduction.

Rotator Cuff Disease

Rotator cuff disease is a spectrum of conditions that ranges from subacromial impingement to rotator cuff tears and eventually to rotator cuff arthropathy (arthritis).

Anatomy The rotator cuff is a group of four muscles that are important in shoulder movements, and maintenance of glenohumeral stability. Scapular Humeral Muscle attachment attachment Action Innervation

Supraspinatus Supraspinatus Superior facet of Initiation of Suprascapular nerve fossa greater tuberosity abduction of humerus

Infraspinatus Infraspinatus Posterior facet of External rotation Suprascapular nerve fossa greater tuberosity of humerus

Teres Minor Lateral border Inferior facet of External rotation Axillary Nerve greater tuberosity of humerus

Subscapularis Subscapular Lesser tuberosity Internal rotation Upper and lower fossa of humerus subscapular nerve

 The inferior rotator cuff muscles (infraspinatus, teres minor, and subscapularis) balance the superior pull of the deltoid. Injury/tear results in upward migration of the humeral head on the glenoid (can be seen on AP radiograph).  Likewise, the anterior muscles (subscapularis) are balanced with the posterior muscles (infraspinatus, teres minor).

Subacromial Impingement

 The most common cause of shoulder pain, which results from impingement of the superior cuff on the undersurface of the acromion, and an inflammatory bursitis.  Associated with certain types of acromial morphology (Bigliani classification).  Presents as insidious pain which is exacerbated by overhead activities.

Rotator Cuff Tear

 Often presents as an acute event on the background of chronic subacromial impingement in the older patient, but can present as an in younger patients.  Majority of tears are to the superior cuff (supraspinatus, infraspinatus, teres minor), though a tear to subscapularis is associated with subcoracoid impingement.  Tears present as pain and weakness when using the muscles in question.

Rotator Cuff Arthropathy

 Defined as shoulder arthritis in the setting of rotator cuff dysfunction. Results from superior migration due to the loss of rotator cuff function and integrity. Unopposed deltoid pulls the humeral head superiorly.  Associated with massive chronic cuff tears.

Imaging

Plain radiographs

 AP of the shoulder may show superior migration of the humerus with a cuff tear, and features of arthritis with arthropathy. Other causes of pain may also be identified (e.g. calcific tendonitis/fracture)  Outlet view is useful for defining the acromial morphology

USS

 Allows dynamic imaging of the cuff, and is inexpensive. However, it is very user dependent.

MRI

 Best imaging modality for cuff pathology.  Also allows imaging of the rest of the shoulder. When intra-articular pathology is suspected, can be combined with an arthrogram for improved sensitivity and specificity.

Treatment

Subacromial impingement

 Physiotherapy, oral anti-inflammatory medication  Subacromial steroid injection can settle inflammation  Arthroscopic subacromial decompression by shaving away the undersurface of the acromion, more space is created for the rotator cuff. Cuff integrity is assessed also at time of surgery, and can be repaired if necessary.

Rotator cuff tear

 When considering repair of a cuff tear, the age and activity of the patient, the nature of the tear (degenerative vs. acute traumatic), and the size and retraction of the tear should be considered when making a surgical plan.  Mild tears or tears in the elderly can be managed conservatively, as outlined above.  Moderate tears can be repaired arthroscopically. Massive or retracted tears will often require an open repair (occasionally with a tendon transfer). Subacromial decompression is performed at the same time to reduce impingement, symptoms and recurrence.

Calcific tendonitis Calcific tendonitis involves calcific deposits within tendons anywhere in the body, but most commonly in the rotator cuff (specifically the supraspinatus tendon). When present in the shoulder, it is associated with subacromial impingement and pain.

Pathology

 More common in women aged 30-60 years.  Association with diabetes and hypothyroidism

There are three stages of calcification

 Formative phase characterized by calcific deposits  Resting phase deposit is stable, but presents with impingement problems  Resorptive phase phagocytic resorption. Most painful stage.

Presentation

 Similar in presentation to subacromial impingement, with pain especially with over head activities. Atraumatic in nature.

Imaging

 Plain radiographs show calcification of the rotator cuff, usually within 1.5cm of its insertion on the humerus. Supraspinatus outlet views can show level of impingment. Further imaging is rarely needed.

Treatment

 Non-operative NSAIDS, steroid injection (controversial, but practiced) and physiotherapy. Approximately 75% will resolve by 6 months with conservative management.  Ultrasound guided or surgical needle barbotage can break down deposits and resolve symptoms. Occasionally surgical excision is required.

Adhesive capsulitis (Frozen Shoulder)

 Pain and loss of movement of shoulder joint, which involves fibroplastic proliferation of capsular tissue, causing soft tissue scarring and contracture. Patients present with a painful and decreased arc of motion.  Associated with prolonged immobilization, previous surgery, thyroid disorders (AI) and diabetes  Classically three stages which can take up to two years to resolve:

Stage one the freezing and painful stage Stage two the frozen and stiff stage Stage three the thawing stage, where shoulder movement slowly improves

Imaging

 Plain radiographs to exclude other causes of a painful shoulder  MRI arthrogram may show capsular contracture, and again may be used to exclude cuff pathology. However, often not performed as diagnosis is largely clinical.

Treatment

 Non-operative NSAIDS, steroid injection and physiotherapy. Patience is required as condition can take up to 2 years to improve.  Operative MUA or arthroscopic adhesiolysis (release of adhesions) can expedite recovery, followed by intensive physiotherapy.

Glenohumeral Arthritis Shoulder arthritis presents with the normal symptoms of arthritis, however primary osteoarthritis is not as commonly the primary cause as seen in other large joints.. A large proportion of shoulder arthritis is rheumatoid (RA), post traumatic, or secondary to rotator cuff arthropathy (discussed above).

Treatment

 Surgical options include hemiarthroplasty, total shoulder replacement and reverse geometry total shoulder replacement (used when rotator cuff function is absent).

References 1. JAMA. 2015;313(10):1037-1047. doi:10.1001/jama.2015.1629 Next question

A 44 year old man is involved in a road traffic accident. He suffers significant injuries to his thorax, he has bilateral haemopneumothoraces and a suspected haemopericardium. He is to undergo surgery, what is the best method of accessing these injuries?

Bilateral thoracoscopy and mediastinoscopy

Midline sternotomy

Bilateral posterolateral thoracotomy

Clam shell thoracotomy

None of the above

Patients with significant mediastinal and lung injuries are best operated on using a Clam shell thoracotomy. All modes of access involve a degree of compromise. A sternotomy would give good access to the heart. However, it takes longer to perform and does not provide good access to the lungs. Trauma should not be managed using laparoscopy. Please rate this question:

Discuss and give feedback Next question Trauma management

The cornerstone of trauma management is embodied in the principles of ATLS.

Following trauma there is a trimodal death distribution:

 Immediately following injury. Typically as result of brain or high spinal injuries, cardiac or great vessel damage. Salvage rate is low.  In early hours following injury. In this group deaths are due to phenomena such as splenic rupture, sub dural haematomas and haemopneumothoraces  In the days following injury. Usually due to sepsis or multi organ failure.

Aspects of trauma management

 ABCDE approach.  Tension pneumothoraces will deteriorate with vigorous ventilation attempts.  External haemorrhage is managed as part of the primary survey. As a rule tourniquets should not be used. Blind application of clamps will tend to damage surrounding structures and packing is the preferred method of haemorrhage control.  Urinary catheters and naso gastric tubes may need inserting. Be wary of basal skull fractures and urethral injuries.  Patients with head and neck trauma should be assumed to have a cervical spine injury until proven otherwise.

Thoracic injuries

 Simple pneumothorax  Mediastinal traversing wounds  Tracheobronchial tree injury  Haemothorax  Blunt cardiac injury  Diaphragmatic injury  Aortic disruption  Pulmonary contusion

Management of thoracic trauma

 Simple pneumothorax insert chest drain. Aspiration is risky in trauma as pneumothorax may be from lung laceration and convert to tension pneumothorax.  Mediastinal traversing wounds These result from situations like stabbings. Exit and entry wounds in separate hemithoraces. The presence of a mediastinal haematoma indicates the likelihood of a great vessel injury. All patients should undergo CT angiogram and oesophageal contrast swallow. Indications for thoracotomy are largely related to blood loss and will be addressed below.  Tracheobronchial tree injury Unusual injuries. In blunt trauma most injuries occur within 4cm of the carina. Features suggesting this injury include haemoptysis and surgical emphysema. These injuries have a very large air leak and may have tension pneumothorax.  Haemothorax Usually caused by laceration of lung vessel or internal mammary artery by rib fracture. Patients should all have a wide bore 36F chest drain. Indications for thoracotomy include loss of more than 1.5L blood initially or ongoing losses of >200ml per hour for >2 hours.  Cardiac contusions Usually cardiac arrhythmias, often overlying sternal fracture. Perform echocardiography to exclude pericardial effusions and tamponade. Risk of arrhythmias falls after 24 hours.  Diaphragmatic injury Usually left sided. Direct surgical repair is performed.  Traumatic aortic disruption Commonest cause of death after RTA or falls. Usually incomplete laceration near ligamentum arteriosum. All survivors will have contained haematoma. Only 1-2% of patients with this injury will have a normal chest x-ray.  Pulmonary contusion Common and lethal. Insidious onset. Early intubation and ventilation.

Abdominal trauma

 Deceleration injuries are common.  In blunt trauma requiring laparotomy the spleen is most commonly injured (40%)  Stab wounds traverse structures most commonly liver (40%)  Gunshot wounds have variable effects depending upon bullet type. Small bowel is most commonly injured (50%)  Patients with stab wounds and no peritoneal signs up to 25% will not enter the peritoneal cavity  Blood at urethral meatus suggests a urethral tear  High riding prostate on PR = urethral disruption  Mechanical testing for pelvic stability should only be performed once

Investigations in abdominal trauma

Diagnostic Peritoneal Abdominal CT scan USS Lavage

Indication Document bleeding if Document organ injury if Document fluid if hypotensive normotensive hypotensive

Advantages Early diagnosis and Most specific for Early diagnosis, non sensitive; 98% accurate localising injury; 92 to invasive and repeatable; 98% accurate 86 to 95% accurate

Disadvantages Invasive and may miss Location of scanner away Operator dependent and retroperitoneal and from facilities, time taken may miss diaphragmatic injury for reporting, need for retroperitoneal injury contrast

 Amylase may be normal following pancreatic trauma  Urethrography if suspected urethral injury

Next question

A 63 year old male is admitted to the surgical ward for an elective femoral popliteal bypass. He suddenly starts complaining of central, crushing chest pain which is radiating to the left arm. The Nursing staff start high flow oxygen and give a spray of glyceryl trinitrate spray. Unfortunately there is no relief of symptoms. What is the next agent to be administered?

Aspirin 75mg

Clopidogrel 75mg

Aspirin 300mg

Clopidogrel 300mg

Direct admission to angiography suite

Aspirin 300mg should be given as soon as possible. If the patient has a moderate to high risk of myocardial infarction, then Clopidogrel should be given with a low molecular weight heparin. Thromboloysis or urgent percutaneous intervention should be given if there are significant ECG changes.

Please rate this question:

Discuss and give feedback

Next question

Management of acute coronary syndrome

NICE produced guidelines in 2010 on the management of unstable angina and non-ST elevation myocardial infarction (NSTEMI). They advocate managing patients based on the early risk assessment using a recognised scoring system such as GRACE (Global Registry of Acute Cardiac Events) to calculate a predicted 6 month mortality.

All patients should receive - aspirin 300mg - nitrates or morphine to relieve chest pain if required

Whilst it is common that non-hypoxic patients receive oxygen therapy there is little evidence to support this approach. The 2008 British Thoracic Society oxygen therapy guidelines advise not giving oxygen unless the patient is hypoxic.

Antithrombin treatment. Low molecular weight heparin should be offered to patients who are not at a high risk of bleeding and who are not having angiography within the next 24 hours. If angiography is likely within 24 hours or a patients creatinine is > 265 umol/l unfractionated heparin should be given.

Clopidogrel 300mg should be given to patients with a predicted 6 month mortality of more than 1.5% or patients who may undergo percutaneous coronary intervention within 24 hours of admission to hospital. Clopidogrel should be continued for 12 months.

Intravenous glycoprotein IIb/IIIa receptor antagonists (eptifibatide or tirofiban) should be given to patients who have an intermediate or higher risk of adverse cardiovascular events (predicted 6- month mortality above 3.0%), and who are scheduled to undergo angiography within 96 hours of hospital admission.

Coronary angiography should be considered within 96 hours of first admission to hospital to patients who have a predicted 6-month mortality above 3.0%. It should also be performed as soon as possible in patients who are clinically unstable.

Next question

Which of the following is not a change found on an ECG in acute pulmonary embolism?

No changes

J waves

P pulmonale

Right ventricular strain

T wave inversion in the inferior leads

S1, Q3, T3

J waves are pathognomonic of hypothermia. Please rate this question:

Discuss and give feedback Next question Pulmonary Embolism: ECG changes

 No changes  S1, Q3, T3  Tall R waves: V1  P pulmonale (peaked P waves): inferior leads  Right axis deviation, Right bundle branch block  Atrial arrhythmias  T wave inversion: V1, V2, V3  Right ventricular strain: if identified is associated with adverse short-term outcome and adds prognostic value to echocardiographic evidence of right ventricular dysfunction in patients with acute pulmonary embolism and normal blood pressure.

References Vanni S et al. Prognostic value of ECG among patients with acute pulmonary embolism and normal blood pressure. Am J Med. 2009 Mar;122(3):257-64. Next question

A 21 year old man falls down a ravine whilst skiing and is trapped for several hours. He is finally brought to the emergency department profoundly hypothermic with a core temperature of 29oC. Which method is most effective at raising the core temperature?

Re-warming with electric blankets

Increasing the room temperature

Instillation of warm intravesical fluid

Instillation of warmed rectal fluid

Instillation of warmed intra peritoneal fluid

Theme from 2011 Examination Visceral cavity re-warming be it lung or abdomen (or both) provides rapid rewarming. Only extracorporeal circulatory devices provide faster rates of re-warming.

Please rate this question:

Discuss and give feedback

Next question

Hypothermia

Core body temperature below 35oC. Severe hypothermia is present when the core temperature is below 30oC. Hypothermia is associated with a reduction in both respiratory and cardiac activity.

Management An organised cardiac rhythm may be converted to fibrillation if CPR is attempted inappropriately so ECG should be analysed with care. The rewarming technique used depends upon the degree of hypothermia and the physiological state of the patient. Mild hypothermia may respond to external rewarming devices. Severe hypothermia may require active core rewarming techniques such as peritoneal lavage, haemodialysis or cardiac bypass. Patients who develop cardiac arrhythmias who are severely hypothermic may respond to bretylium toslyte (sadly no longer available in most centres), but do not generally respond to standard therapies or DC cardioversion.

Next question

A 42 year old woman is admitted to surgery with acute cholecystitis. She is known to have hypertension, rheumatoid arthritis and polymyalgia rheumatica. Her medical therapy includes: Paracetamol 1g qds Codeine phosphate 30mg qds Bendrofluazide 2.5 mg od Ramipril 10mg od Methotrexate 7.5mg once a week Prednisolone 5mg od You are called by the Senior House Officer to assess this lady as she has become delirious and hypotensive 2 hours after surgery. Her blood results reveal:

Na+ 132 mmol/l

K+ 5.3 mmol/l

Urea 7 mmol/l

Creatinine 108 µmol/l

Hb 12.4 g/dl

Platelets 178 * 109/l

WBC 15.4 * 109/l

What management is needed immediately?

Ceftriaxone IV

Hydrocortisone 50mg IV

CT scan abdomen

Urgent exploratory laparotomy

Hydrocortisone 100mg IV

This patient has acute adrenal insufficiency and urgently needs steroid replacement. Please rate this question:

Discuss and give feedback Next question Addisonian crisis

Causes

 Sepsis or surgery causing an acute exacerbation of chronic insufficiency (Addison's, Hypopituitarism)  Adrenal haemorrhage eg Waterhouse-Friderichsen syndrome (fulminant meningococcemia)  Steroid withdrawal

Management

 Hydrocortisone 100 mg im or iv  1 litre normal saline infused over 30-60 mins or with dextrose if hypoglycaemic  Continue hydrocortisone 6 hourly until the patient is stable. No fludrocortisone is required because high cortisol exerts weak mineralocorticoid action  Oral replacement may begin after 24 hours and be reduced to maintenance over 3-4 days

Next question

Theme: Thoracic trauma

A. Thoracotomy B. Manage conservatively C. Intercostal tube drain insertion D. CT scanning E. Bronchoscopy F. Negative pressure intercostal tube drainage G. Video assisted thoracoscopy and pleurectomy

For each of the following scenarios please select the most appropriate management option from the list. Each option may be used once, more than once or not at all.

54. A 28 year old male is involved in a road traffic accident he is thrown from his motorbike onto the pavement and sustains a haemopneumothorax and flail segment of the right chest.

You answered Thoracotomy

The correct answer is Intercostal tube drain insertion

He requires a chest drain and analgesia. In general all haemopneumothoraces should be managed by intercostal chest drain insertion as they have a risk of becoming a tension pneumothorax until the lung laceration has sealed.

55. A 19 year old man is stabbed in the chest at a nightclub. He develops a cardiac arrest in casualty following an attempted transfer to the CT scanning room.

Thoracotomy

This is one indication for an 'emergency room' thoracotomy, there are not many others! Typical injuries include ventricular penetration, great vessel disruption and hilar lung injuries.

56. A 32 year old male falls over and sustains a small pneumothorax following a simple rib fracture. He has no physiological compromise.

You answered Thoracotomy

The correct answer is Intercostal tube drain insertion

Unlike spontaneous pneumothoraces most would advocate chest tube drainage in the context of pneumothorax resulting from trauma. This is because of the risk of the lung laceration developing a tension. Once there is no further evidence of air leak the chest drain may be removed and a check x-ray performed to check there is no re-accumulation prior to discharge.

Please rate this question:

Discuss and give feedback Next question Thoracic trauma

Types of thoracic trauma

Tension  Often laceration to lung parenchyma with flap pneumothorax  Pressure develops in thorax  Most common cause is mechanical ventilation in patient with pleural injury  Symptoms overlap with cardiac tamponade, hyper-resonant percussion note is more likely in tension pnemothorax

Flail chest  Chest wall disconnects from thoracic cage  Multiple rib fractures (at least two fractures per rib in at least two ribs)  Associated with pulmonary contusion  Abnormal chest motion  Avoid over hydration and fluid overload

Pneumothorax  Most common cause is lung laceration with air leakage  Most traumatic pneumothoraces should have a chest drain  Patients with traumatic pneumothorax should never be mechanically ventilated until a chest drain is inserted

Haemothorax  Most commonly due to laceration of lung, intercostal vessel or internal mammary artery  Haemothoraces large enough to appear on CXR are treated with large bore chest drain  Surgical exploration is warranted if >1500ml blood drained immediately

Cardiac tamponade  Beck's triad: elevated venous pressure, reduced arterial pressure, reduced heart sounds  Pulsus paradoxus  May occur with as little as 100ml blood

Pulmonary contusion  Most common potentially lethal chest injury  Arterial blood gases and pulse oximetry important  Early intubation within an hour if significant hypoxia

Blunt cardiac injury  Usually occurs secondary to chest wall injury  ECG may show features of myocardial infarction  Sequelae: hypotension, arrhythmias, cardiac wall motion abnormalities

Aorta disruption  Deceleration injuries  Contained haematoma  Widened mediastinum

Diaphragm  Most due to motor vehicle accidents and blunt trauma causing disruption large radial tears (laceration injuries result in small tears)  More common on left side  Insert gastric tube, may pass into intrathoracic stomach

Mediastinal  Entrance wound in one hemithorax and exit wound/foreign body traversing wounds in opposite hemithorax  Mediastinal haematoma or pleural cap suggests great vessel injury  Mortality is 20%

References ATLS Manual 8th Edition Next question

A 21 year old man is undergoing an inguinal hernia repair and receives a dose of intravenous co- amoxiclav. He is reported to have a penicillin allergy. Over the next few minutes his vital signs are: Pulse - 130bpm, blood pressure- 60/40mmHg. What is the first line treatment?

Hydrocortisone 100mg IV

Adrenaline 1:1000 IV

Chlorpheniramine 10mg IV

Adrenaline 1:1000 IM

Adrenaline 1:10000 IV

Theme from 2009 Exam Theme from September 2013 Exam The first line treatment of anaphylactic shock is intra muscular adrenaline. Please rate this question:

Discuss and give feedback Next question Anaphylactic shock

 Suspect if there has been exposure to an allergen

Management - Remove allergen - ABCD - Drugs:

Adrenaline 1:1000 0.5ml INTRAMUSCULARLY (not IV). Repeat after 5 mins if no response. Then Chlorpheniramine 10mg IV Then Hydrocortisone 100-200mg IV

Reference Emergency treatment of anaphylactic reactions. Guidelines for healthcare providers. Working Group of the Resuscitation Council (UK).2008 Next question

Theme: Types of stroke

A. Anterior cerebral artery infarct B. Middle cerebral artery infarct C. Posterior cerebral artery infarct D. Pituitary mass E. Lacunar infarct F. Lateral medullary syndrome G. Pontine infarct H. Horner's syndrome I. Cerebellar infarct

Please select the most likely cause for the symptoms given. Each option may be used once, more than once or not at all.

58. A 53 year old teacher is admitted to the vascular ward for a carotid endarterectomy. Your house officer does a preoperative assessment and notes that there is a right homonymous hemianopia. There is no other neurology.

You answered Anterior cerebral artery infarct

The correct answer is Posterior cerebral artery infarct

This patient has had a left occipital infarct, as there is only a homonymous hemianopia. If this patient had a temporal or parietal lobe infarct, there would be associated hemiparesis and higher cortical dysfunction. This is important to differentiate, as the carotid endarterectomy is inappropriate in this patient as the lesion is in the posterior cerebral artery.

59. A 52 year man is admitted to the vascular ward for an amputation. The patient complains of unsteadiness. On further examination you detect right facial numbness and right sided nystagmus. There is sensory loss of the left side of the body.

You answered Anterior cerebral artery infarct

The correct answer is Lateral medullary syndrome

A combination of ipsilateral ataxia, nystagmus, dysphagia, facial numbness, cranial nerve palsy with contralateral hemisensory loss indicates this diagnosis.

60. A 48 year old type 2 diabetic complains of numbness in his left arm and leg. Otherwise there is no other neurological signs.

You answered Anterior cerebral artery infarct

The correct answer is Lacunar infarct

Isolated hemisensory loss is a feature of a lacunar infarct.

Please rate this question:

Discuss and give feedback Next question Stroke: types

Primary intracerebral  Presents with headache, vomiting, loss of haemorrhage (PICH, c. 10%) consciousness

Total anterior circulation  Involves middle and anterior cerebral arteries infarcts (TACI, c. 15%)  Hemiparesis/hemisensory loss  Homonymous hemianopia  Higher cognitive dysfunction e.g. Dysphasia

Partial anterior circulation  Involves smaller arteries of anterior circulation e.g. infarcts (PACI, c. 25%) upper or lower division of middle cerebral artery  Higher cognitive dysfunction or two of the three TACI features

Lacunar infarcts (LACI, c.  Involves perforating arteries around the internal 25%) capsule, thalamus and basal ganglia  Present with either isolated hemiparesis, hemisensory loss or hemiparesis with limb ataxia

Posterior circulation infarcts  Vertebrobasilar arteries (POCI, c. 25%)  Presents with features of brainstem damage  Ataxia, disorders of gaze and vision, cranial nerve lesions

Lateral medullary syndrome  Wallenberg's syndrome (posterior inferior cerebellar  Ipsilateral: ataxia, nystagmus, dysphagia, facial artery) numbness, cranial nerve palsy  Contralateral: limb sensory loss

Weber's syndrome  Ipsilateral III palsy  Contralateral weakness

------

Anterior cerebral artery

 Contralateral hemiparesis and sensory loss, lower extremity > upper  Disconnection syndrome

Middle cerebral artery

 Contralateral hemiparesis and sensory loss, upper extremity > lower  Contralateral hemianopia  Aphasia (Wernicke's)  Gaze abnormalities

Posterior cerebral artery

 Contralateral hemianopia with macular sparing  Disconnection syndrome

Lacunar

 Present with either isolated hemiparesis, hemisensory loss or hemiparesis with limb ataxia

Lateral medulla (posterior inferior cerebellar artery)

 Ipsilateral: ataxia, nystagmus, dysphagia, facial numbness, cranial nerve palsy e.g.

Horner's

 Contralateral: limb sensory loss

Pontine

 VI nerve: horizontal gaze palsy  VII nerve  Contralateral hemiparesis

Next question

A 22 year old man has a full thickness burn on his chest. It is well circumscribed. In A&E his saturations are reduced to 92% on 15L Oxygen, Blood pressure 102/66 mmHg and HR 105bpm. What is the best management?

Haemodialysis

Escharotomy

Fasciotomy

Cardiac bypass

Non invasive ventilation

The chest burn and its associated oedema is limiting respiration. Therefore an escharotomy of the chest is indicated, this will remove the constriction on the chest wall and improve ventilation. Please rate this question:

Discuss and give feedback Next question Burns

Burns may be thermal, chemical or electrical. In the former category are burns which occur as a result of heat. Chemical burns occur when the skin is exposed to an extremely caustic or alkaline substance. Electrical burns occur following exposure to electrical current. The immediate management includes removal of the burning source which usually includes irrigation of the burned area. A detailed assessment then needs to be made of the extent of the burns and a number of charts are available for recording this information. The degree of injury relates to the temperature and duration of exposure. Most domestic burns are mainly scalds in young children.

Following the burn, there is a local response with progressive tissue loss and release of inflammatory cytokines. Systemically, there are cardiovascular effects resulting from fluid loss and sequestration of fluid into the third space. There is a marked catabolic response. Immunosupression is common with large burns and bacterial translocation from the gut lumen is a recognised event. Sepsis is a common cause of death following major burns.

Types of burn

Type of burn Skin layers Skin Blanching Management affected appearance

Epidermal/Superficial Epidermis Red, moist Yes

Superficial partial Epidermis and part Pale, dry Yes Normally heals thickness of papillary dermis with no affected intervention

Deep partial thickness Epidermis, whole Mottled red No Needs surgical papillary dermis colour intervention affected (depending on site)

Full thickness Whole skin layer Dry, leathery No Burns centre and subcutaneous hard wound tissue affected

Depth of burn assessment

 Bleeding on needle prick  Sensation  Appearance  Blanching to pressure

Percentage burn estimation Lund Browder chart: most accurate even in children Wallace rule of nines Palmar surface: surface area palm = 0.8% burn

>15% body surface area burns in adults needs urgent burn fluid resuscitation

Transfer to burn centre if:

 Need burn shock resuscitation  Face/hands/genitals affected  Deep partial thickness or full thickness burns  Significant electrical/chemical burns

Management The initial aim is to stop the burning process and resuscitate the patient. Intravenous fluids will be required for children with burns greater than 10% of total body surface area. Adults with burns greater than 15% of total body surface area will also require IV fluids. The fluids are calculated using the Parkland formula which is; volume of fluid= total body surface area of the burn % x weight (Kg) x4. Half of the fluid is administered in the first 8 hours. A urinary catheter should be inserted. Analgesia should be given. Complex burns, burns involving the hand perineum and face and burns >10% in adults and >5% in children should be transferred to a burns unit.

Circumferential burns affecting a limb or severe torso burns impeding respiration may require escharotomy to divide the burnt tissue.

Conservative management is appropriate for superficial burns and mixed superficial burns that will heal in 2 weeks. More complex burns may require excision and skin grafting. Excision and primary closure is not generally practised as there is a high risk of infection.

There is no evidence to support the use of anti microbial prophylaxis or topical antibiotics in burn patients.

Escharotomies

 Indicated in circumferential full thickness burns to the torso or limbs.  Careful division of the encasing band of burn tissue will potentially improve ventilation (if the burn involves the torso), or relieve compartment syndrome and oedema (where a limb is involved)

References www.euroburn.org/e107files/downloads/guidelinesburncare.pdf

Barajas-Nava LA, López-Alcalde J, Roqué i Figuls M, Solà I, Bonfill Cosp X. Antibiotic prophylaxis for preventing burn wound infection. Cochrane Database of Systematic Reviews 2013, Issue 6. Art. No.: CD008738. DOI: 10.1002/14651858.CD008738.pub2.

Hettiaratchy S & Papini R. Initial management of a major burn: assessment and resuscitation. BMJ 2004;329:101-103 Next question

A 16 year old man sustains a basal skull fracture and is suspected of having CSF rhinorrhoea. Which of the following laboratory tests would most accurately identify whether CSF is present or not?

Microscopy to identify red blood cells

Lab stix testing for glucose

Lab stix testing for protein

Beta 2 transferrin assay

Microscopy, gram stain and culture

Beta 2 transferrin is a carbohydrate free form of transferrin that is almost exclusively found in the CSF. Although lab stix testing for glucose is traditional it is associated with false positive results secondary to contamination with other glucose containing bodily secretions. Please rate this question:

Discuss and give feedback Next question Head injury management- NICE Guidelines

Summary of guidelines

 All patients should be assessed within 15 minutes on arrival to A&E  Document all 3 components of the GCS  If GCS <8 or = to 8, consider stabilising the airway  Treat pain with low dose IV opiates (if safe)  Full spine immobilisation until assessment if:

- GCS < 15 - neck pain/tenderness - paraesthesia extremities - focal neurological deficit - suspected c-spine injury

If a c-spine injury is suspected a 3 view c-spine x-ray is indicated. CT c-spine is preferred if: - Intubated - GCS <13 - Normal x-ray but continued concerns regarding c-spine injury - Any focal neurology - A CT head scan is being performed - Initial plain films are abnormal

Immediate CT head (within 1 hour) if:

 GCS < 13 on admission  GCS < 15 2 hours after admission  Suspected open or depressed skull fracture  Suspected skull base fracture (panda eyes, Battle's sign, CSF from nose/ear, bleeding ear)  Focal neurology  Vomiting > 1 episode  Post traumatic seizure  Coagulopathy

Contact neurosurgeon if:

 Persistent GCS < 8 or = 8  Unexplained confusion > 4h  Reduced GCS after admission  Progressive neurological signs  Incomplete recovery post seizure  Penetrating injury  Cerebrospinal fluid leak

Observations

 1/2 hourly GCS until 15

Reference 1. http://guidance.nice.org.uk/CG56/QuickRefGuide/pdf/English 2. Hodgkinson S et al. Early management of head injury: summary of NICE guidance. BMJ 2014 (348):34-37. Next question

A 66 year old male is admitted to the vascular ward for an amputation. He reports episodes of vertigo and dysarthria to the house officer. He suddenly collapses with a Glasgow Coma Score of 3. What is the most likely diagnosis?

Cerebral haemorrhage in left temporal parietal area

Opiate overdose

Cerebral haemorrhage in right temporal parietal area

Diazepam overdose

Basilar artery occlusion

Vertigo and dysarthria suggest a posterior circulation event. In the scenario of a patient complaining of posterior symptoms and a sudden deterioration in consciousness, the main differential diagnosis is of a basilar artery occlusion. Please rate this question:

Discuss and give feedback Next question Stroke: types

Primary intracerebral  Presents with headache, vomiting, loss of haemorrhage (PICH, c. 10%) consciousness

Total anterior circulation  Involves middle and anterior cerebral arteries infarcts (TACI, c. 15%)  Hemiparesis/hemisensory loss  Homonymous hemianopia  Higher cognitive dysfunction e.g. Dysphasia

Partial anterior circulation  Involves smaller arteries of anterior circulation e.g. infarcts (PACI, c. 25%) upper or lower division of middle cerebral artery  Higher cognitive dysfunction or two of the three TACI features Lacunar infarcts (LACI, c.  Involves perforating arteries around the internal 25%) capsule, thalamus and basal ganglia  Present with either isolated hemiparesis, hemisensory loss or hemiparesis with limb ataxia

Posterior circulation infarcts  Vertebrobasilar arteries (POCI, c. 25%)  Presents with features of brainstem damage  Ataxia, disorders of gaze and vision, cranial nerve lesions

Lateral medullary syndrome  Wallenberg's syndrome (posterior inferior cerebellar  Ipsilateral: ataxia, nystagmus, dysphagia, facial artery) numbness, cranial nerve palsy  Contralateral: limb sensory loss

Weber's syndrome  Ipsilateral III palsy  Contralateral weakness

------

Anterior cerebral artery

 Contralateral hemiparesis and sensory loss, lower extremity > upper  Disconnection syndrome

Middle cerebral artery

 Contralateral hemiparesis and sensory loss, upper extremity > lower  Contralateral hemianopia  Aphasia (Wernicke's)  Gaze abnormalities

Posterior cerebral artery

 Contralateral hemianopia with macular sparing  Disconnection syndrome

Lacunar

 Present with either isolated hemiparesis, hemisensory loss or hemiparesis with limb ataxia

Lateral medulla (posterior inferior cerebellar artery)

 Ipsilateral: ataxia, nystagmus, dysphagia, facial numbness, cranial nerve palsy e.g.

Horner's

 Contralateral: limb sensory loss

Pontine

 VI nerve: horizontal gaze palsy  VII nerve  Contralateral hemiparesis

Next question

A 22 year old man is brought to the emergency department. He was found lying unconscious on his right arm and it is evident that he has taken a temazepam overdose. His right arm is mottled in colour and swollen, his hand is insensate and stiff. What substance is most likely to be present in the urine in increased quantities?

Protein

Haemoglobin

Myoglobin

Erythrocytes

Lymphocytes

Theme from 2011 Exam This man is likely to have muscle death secondary to compartment syndrome. This will result in muscle breakdown and release of myoglobin. This may accumulate in the kidney and result in renal failure. Please rate this question:

Discuss and give feedback Next question Compartment syndrome

 This is a particular complication that may occur following fractures (or following ischaemia re- perfusion injury in vascular patients). It is characterised by raised pressure within a closed anatomical space.  The raised pressure within the compartment will eventually compromise tissue perfusion resulting in necrosis. The two main fractures carrying this complication include supracondylar fractures and tibial shaft injuries.

Symptoms and signs

 Pain, especially on movement (even passive)  Parasthesiae  Pallor may be present  Arterial pulsation may still be felt as the necrosis occurs as a result of microvascular compromise  Paralysis of the muscle group may occur

Diagnosis

 Is made by measurement of intracompartmental pressure measurements. Pressures in excess of 20mmHg are abnormal and >40mmHg is diagnostic.

Treatment

 This is essentially prompt and extensive fasciotomies  In the lower limb the deep muscles may be inadequately decompressed by the inexperienced operator when smaller incisions are performed  Myoglobinuria may occur following fasciotomy and result in renal failure and for this reason these patients require aggressive IV fluids  Where muscle groups are frankly necrotic at fasciotomy they should be debrided and amputation may have to be considered  Death of muscle groups may occur within 4-6 hours

Next question

A 19 year intravenous drug abuser is recovering following a surgical drainage of a psoas abscess. He is found collapsed in the ward toilet unresponsive and with pinpoint pupils. What is the most appropriate immediate management?

Intravenous flumazenil

Intravenous nalaxone

Intravenous benxhexol

No further management

Intravenous glycopyrolate

Intravenous nalaxone is needed to treat the patient who has had an overdose of opiate. Naloxone has the quickest onset of action, however it is important to be aware of its short acting duration and the need for further administration. There is also the risk of rebound pain once naloxone is given. Please rate this question:

Discuss and give feedback Next question Opioid misuse

Opioids are substances which bind to opioid receptors. This includes both naturally occurring opiates such as morphine and synthetic opioids such as buprenorphine and methadone.

Features of opioid misuse

 Rhinorrhoea  Needle track marks  Pinpoint pupils  Drowsiness

Complications of intravenous opioid misuse

 Viral infection secondary to sharing needles: HIV, hepatitis B & C  Bacterial infection secondary to injection: infective endocarditis, septic arthritis, septicaemia, necrotising fasciitis, groin abscess  Pseudoaneurysm  Venous thromboembolism  Osteomyelitis  Overdose may lead to respiratory depression and death

Emergency management of opioid overdose

 IV or IM naloxone: has a rapid onset and relatively short duration of action

Next question

A 68 year old male is admitted to the surgical ward for assessment of severe epigastric pain. His abdomen is soft and non tender. However the Nurse forces you to look at the ECG. It looks abnormal. Which of the following features is an indication for urgent coronary thrombolysis or percutaneous intervention?

Right bundle branch block

ST elevation of 1mm in leads V1 to V6

Ventricular tachycardia

Q waves in leads V1 to V6

ST elevation of 1mm in leads II, III and aVF

ECG changes for thrombolysis or percutaneous intervention: ST elevation of > 2mm (2 small squares) in 2 or more consecutive anterior leads (V1-V6) OR

ST elevation of greater than 1mm (1 small square) in greater than 2 consecutive inferior leads (II, III, avF, avL) OR

New Left bundle branch block

ST elevation of 1mm in leads II, III and aVF reflects significant cardiac ischaemia due to the right coronary artery occlusion. The medical registrar should be contacted to urgently assess the patient. Note right coronary artery occlusions puts the patient at risk of cardiac arrhythmias (due to blood supply to the sino atrial node). Please rate this question:

Discuss and give feedback Next question Thrombolysis or percutaneous intervention in myocardial infarction

Thrombolytic drugs activate plasminogen to form plasmin. This in turn degrades fibrin and help breaks up thrombi. They in primarily used in patients who present with a ST elevation myocardial infarction. Other indications include acute ischaemic stroke and pulmonary embolism, although strict inclusion criteria apply.

Examples

 alteplase  tenecteplase  streptokinase

Contraindications to thrombolysis

 active  recent haemorrhage, trauma or surgery (including dental extraction)  coagulation and bleeding disorders  intracranial neoplasm  stroke < 3 months  aortic dissection  recent head injury  pregnancy  severe hypertension

Side-effects

 haemorrhage  hypotension - more common with streptokinase  allergic reactions may occur with streptokinase

Next question

Which of the following is not typically associated with a injury?

Overlying pallor of the skin

Abnormal motility of the overlying skin

History of friction type injury

Improved results when the degloved segment is left in situ as a temporary closure

Poor results when primary compression treatment is used in preference to skin grafting

Degloving injuries typically involve extremities and are usually friction injuries e.g. arm being run over. There is abnormal motility of the overlying skin, pallor, loss of sensation. Early treatment is key and should involve skin grafting which may use the degloved segment. This however, should be formally prepared for the role and simple compression bandaging gives poor results. Please rate this question:

Discuss and give feedback Next question

Surgical wounds are either incisional or excisional and either clean, clean contaminated or dirty. Although the stages of wound healing are broadly similar their contributions will vary according to the wound type.

The main stages of wound healing include:

Haemostasis

 Minutes to hours following injury  Vasospasm in adjacent vessels, platelet plug formation and generation of fibrin rich clot.

Inflammation

 Typically days 1-5  Neutrophils migrate into wound (function impaired in diabetes).  Growth factors released, including basic fibroblast growth factor and vascular endothelial growth factor.  Fibroblasts replicate within the adjacent matrix and migrate into wound.  Macrophages and fibroblasts couple matrix regeneration and clot substitution.

Regeneration

 Typically days 7 to 56  Platelet derived growth factor and transformation growth factors stimulate fibroblasts and epithelial cells.  Fibroblasts produce a collagen network.  Angiogenesis occurs and wound resembles granulation tissue.

Remodeling

 From 6 weeks to 1 year  Longest phase of the healing process and may last up to one year (or longer).  During this phase fibroblasts become differentiated (myofibroblasts) and these facilitate wound contraction.  Collagen fibres are remodeled.  Microvessels regress leaving a pale scar.

The above description represents an idealised scenario. A number of diseases may distort this process. Neovascularisation is an important early process. Endothelial cells may proliferate in the wound bed and recanalise to form a vessel. Vascular disease, shock and sepsis can all compromise microvascular flow and impair healing.

Conditions such as jaundice will impair fibroblast synthetic function and immunity with a detrimental effect in most parts of the healing process.

Problems with scars:

Hypertrophic scars Excessive amounts of collagen within a scar. Nodules may be present histologically containing randomly arranged fibrils within and parallel fibres on the surface. The tissue itself is confined to the extent of the wound itself and is usually the result of a full thickness dermal injury. They may go on to develop .

Image of hypertrophic scarring. Note that it remains confined to the boundaries of the original wound:

Image sourced from Wikipedia

Keloid scars Excessive amounts of collagen within a scar. Typically a keloid scar will pass beyond the boundaries of the original injury. They do not contain nodules and may occur following even trivial injury. They do not regress over time and may recur following removal.

Image of a keloid scar. Note the extension beyond the boundaries of the original incision:

Image sourced from Wikipedia

Drugs which impair wound healing:

 Non steroidal anti inflammatory drugs  Steroids  Immunosupressive agents  Anti neoplastic drugs

Closure Delayed primary closure is the anatomically precise closure that is delayed for a few days but before granulation tissue becomes macroscopically evident.

Secondary closure refers to either spontaneous closure or to surgical closure after granulation tissue has formed. Next question

Which of the following statements relating to large volume blood loss in trauma is incorrect?

Tranexamic acid reduces the incidence of rebleeding following surgery

Hypocalcaemia may complicate resuscitation

Colloids are preferred initially as they reduce the incidence of coagulopathy

When patients receive over 5 units of whole blood mortality increases when blood

products greater than 3 weeks old are utilised

In the battlefield setting a ratio of 1:1:1 for blood, plasma and platelets is used

Fresh blood is the fluid of choice when large volume blood loss complicates trauma. Mortality is doubled when blood >3 weeks old is used. Please rate this question:

Discuss and give feedback Next question Trauma management

The cornerstone of trauma management is embodied in the principles of ATLS.

Following trauma there is a trimodal death distribution:

 Immediately following injury. Typically as result of brain or high spinal injuries, cardiac or great vessel damage. Salvage rate is low.  In early hours following injury. In this group deaths are due to phenomena such as splenic rupture, sub dural haematomas and haemopneumothoraces  In the days following injury. Usually due to sepsis or multi organ failure.

Aspects of trauma management

 ABCDE approach.  Tension pneumothoraces will deteriorate with vigorous ventilation attempts.  External haemorrhage is managed as part of the primary survey. As a rule tourniquets should not be used. Blind application of clamps will tend to damage surrounding structures and packing is the preferred method of haemorrhage control.  Urinary catheters and naso gastric tubes may need inserting. Be wary of basal skull fractures and urethral injuries.  Patients with head and neck trauma should be assumed to have a cervical spine injury until proven otherwise.

Thoracic injuries

 Simple pneumothorax  Mediastinal traversing wounds  Tracheobronchial tree injury  Haemothorax  Blunt cardiac injury  Diaphragmatic injury  Aortic disruption  Pulmonary contusion

Management of thoracic trauma

 Simple pneumothorax insert chest drain. Aspiration is risky in trauma as pneumothorax may be from lung laceration and convert to tension pneumothorax.  Mediastinal traversing wounds These result from situations like stabbings. Exit and entry wounds in separate hemithoraces. The presence of a mediastinal haematoma indicates the likelihood of a great vessel injury. All patients should undergo CT angiogram and oesophageal contrast swallow. Indications for thoracotomy are largely related to blood loss and will be addressed below.  Tracheobronchial tree injury Unusual injuries. In blunt trauma most injuries occur within 4cm of the carina. Features suggesting this injury include haemoptysis and surgical emphysema. These injuries have a very large air leak and may have tension pneumothorax.  Haemothorax Usually caused by laceration of lung vessel or internal mammary artery by rib fracture. Patients should all have a wide bore 36F chest drain. Indications for thoracotomy include loss of more than 1.5L blood initially or ongoing losses of >200ml per hour for >2 hours.  Cardiac contusions Usually cardiac arrhythmias, often overlying sternal fracture. Perform echocardiography to exclude pericardial effusions and tamponade. Risk of arrhythmias falls after 24 hours.  Diaphragmatic injury Usually left sided. Direct surgical repair is performed.  Traumatic aortic disruption Commonest cause of death after RTA or falls. Usually incomplete laceration near ligamentum arteriosum. All survivors will have contained haematoma. Only 1-2% of patients with this injury will have a normal chest x-ray.  Pulmonary contusion Common and lethal. Insidious onset. Early intubation and ventilation.

Abdominal trauma

 Deceleration injuries are common.  In blunt trauma requiring laparotomy the spleen is most commonly injured (40%)  Stab wounds traverse structures most commonly liver (40%)  Gunshot wounds have variable effects depending upon bullet type. Small bowel is most commonly injured (50%)  Patients with stab wounds and no peritoneal signs up to 25% will not enter the peritoneal cavity  Blood at urethral meatus suggests a urethral tear  High riding prostate on PR = urethral disruption  Mechanical testing for pelvic stability should only be performed once

Investigations in abdominal trauma

Diagnostic Peritoneal Abdominal CT scan USS Lavage

Indication Document bleeding if Document organ injury if Document fluid if hypotensive normotensive hypotensive

Advantages Early diagnosis and Most specific for Early diagnosis, non sensitive; 98% accurate localising injury; 92 to invasive and repeatable; 98% accurate 86 to 95% accurate

Disadvantages Invasive and may miss Location of scanner away Operator dependent and retroperitoneal and from facilities, time taken may miss diaphragmatic injury for reporting, need for retroperitoneal injury contrast

 Amylase may be normal following pancreatic trauma  Urethrography if suspected urethral injury

Next question

The following features are typical of superficial partial dermal burns except:

Erythema

Absence of

Spontaneous healing in most cases

No extension beyond proximal dermal papillae

Good capillary refill at the burn site

Superficial dermal burns are typically erythematous, do not extend beyond the upper part of the dermal papillae, capillary return and blisters are both usually present.

A Typical example of a superficial dermal burn is shown below

Image sourced from Wikipedia

Please rate this question:

Discuss and give feedback Next question Burns

Burns may be thermal, chemical or electrical. In the former category are burns which occur as a result of heat. Chemical burns occur when the skin is exposed to an extremely caustic or alkaline substance. Electrical burns occur following exposure to electrical current. The immediate management includes removal of the burning source which usually includes irrigation of the burned area. A detailed assessment then needs to be made of the extent of the burns and a number of charts are available for recording this information. The degree of injury relates to the temperature and duration of exposure. Most domestic burns are mainly scalds in young children.

Following the burn, there is a local response with progressive tissue loss and release of inflammatory cytokines. Systemically, there are cardiovascular effects resulting from fluid loss and sequestration of fluid into the third space. There is a marked catabolic response. Immunosupression is common with large burns and bacterial translocation from the gut lumen is a recognised event. Sepsis is a common cause of death following major burns.

Types of burn

Type of burn Skin layers Skin Blanching Management affected appearance

Epidermal/Superficial Epidermis Red, moist Yes

Superficial partial Epidermis and part Pale, dry Yes Normally heals thickness of papillary dermis with no affected intervention

Deep partial thickness Epidermis, whole Mottled red No Needs surgical papillary dermis colour intervention affected (depending on site)

Full thickness Whole skin layer Dry, leathery No Burns centre and subcutaneous hard wound tissue affected

Depth of burn assessment

 Bleeding on needle prick  Sensation  Appearance  Blanching to pressure

Percentage burn estimation Lund Browder chart: most accurate even in children Wallace rule of nines Palmar surface: surface area palm = 0.8% burn

>15% body surface area burns in adults needs urgent burn fluid resuscitation

Transfer to burn centre if:

 Need burn shock resuscitation  Face/hands/genitals affected  Deep partial thickness or full thickness burns  Significant electrical/chemical burns

Management The initial aim is to stop the burning process and resuscitate the patient. Intravenous fluids will be required for children with burns greater than 10% of total body surface area. Adults with burns greater than 15% of total body surface area will also require IV fluids. The fluids are calculated using the Parkland formula which is; volume of fluid= total body surface area of the burn % x weight (Kg) x4. Half of the fluid is administered in the first 8 hours. A urinary catheter should be inserted. Analgesia should be given. Complex burns, burns involving the hand perineum and face and burns >10% in adults and >5% in children should be transferred to a burns unit.

Circumferential burns affecting a limb or severe torso burns impeding respiration may require escharotomy to divide the burnt tissue.

Conservative management is appropriate for superficial burns and mixed superficial burns that will heal in 2 weeks. More complex burns may require excision and skin grafting. Excision and primary closure is not generally practised as there is a high risk of infection.

There is no evidence to support the use of anti microbial prophylaxis or topical antibiotics in burn patients.

Escharotomies

 Indicated in circumferential full thickness burns to the torso or limbs.  Careful division of the encasing band of burn tissue will potentially improve ventilation (if the burn involves the torso), or relieve compartment syndrome and oedema (where a limb is involved)

References www.euroburn.org/e107files/downloads/guidelinesburncare.pdf

Barajas-Nava LA, López-Alcalde J, Roqué i Figuls M, Solà I, Bonfill Cosp X. Antibiotic prophylaxis for preventing burn wound infection. Cochrane Database of Systematic Reviews 2013, Issue 6. Art. No.: CD008738. DOI: 10.1002/14651858.CD008738.pub2.

Which of the following is not a feature found on a CXR in traumatic aortic disruption?

Widened mediastinum

Trachea deviated to the left

Depression of the left main stem bronchus

Obliteration of the aortic knob

Widened paraspinal interfaces

The trachea is normally deviated to the right. Please rate this question:

Discuss and give feedback Next question Thoracic aorta rupture

 Mechanism of injury: Decelerating force i.e. RTA, fall from a great height  Most people die at scene  Survivors may have an incomplete laceration at the ligamentum arteriosum of the aorta.

Clinical features

 Contained haematoma: persistent hypotension  Detected mainly by history, CXR changes

CXR changes

 Widened mediastinum  Trachea/Oesophagus to right  Depression of left main stem bronchus  Widened paratracheal stripe/paraspinal interfaces  Space between aorta and pulmonary artery obliterated  Rib fracture/left haemothorax

Diagnosis Angiography, usually CT aortogram.

Treatment Repair or replacement. Ideally they should undergo endovascular repair. Next question

Theme: Head injury management

A. Observation B. CT head within 1 hour C. CT head within 8 hours D. Urgent neurosurgical review (even before CT head performed) E. 3 view c-spine xray F. 2 view c-spine xray G. CT c-spine H. MRI c-spine

What is the best initial management plan for the scenario given? Each option may be used once, more than once or not at all.

3. A 22 year old male falls of a ladder. He complains of neck pain and cannot feel his legs. His GCS suddenly deteriorates and a CT head confirms an extradural haematoma. What is the best imaging for his neck?

You answered Observation

The correct answer is CT c-spine

This man needs a CT scan of his c-spine. A CT scan will give the best resolution of any bony injury.

4. A 25 year old teacher falls down the stairs. She complains of a headache and has vomited 3 times. She has a GCS of 15/15.

You answered Observation

The correct answer is CT head within 1 hour

This lady has a head injury and vomiting > 1, therefore an urgent CT head is indicated.

5. An 18 year old student is shot in the back of the head.

You answered Observation

The correct answer is Urgent neurosurgical review (even before CT head performed)

A penetrating injury needs urgent neurosurgical review.

Please rate this question:

Discuss and give feedback Next question Head injury management- NICE Guidelines

Summary of guidelines

 All patients should be assessed within 15 minutes on arrival to A&E  Document all 3 components of the GCS  If GCS <8 or = to 8, consider stabilising the airway  Treat pain with low dose IV opiates (if safe)  Full spine immobilisation until assessment if:

- GCS < 15 - neck pain/tenderness - paraesthesia extremities - focal neurological deficit - suspected c-spine injury

If a c-spine injury is suspected a 3 view c-spine x-ray is indicated. CT c-spine is preferred if: - Intubated - GCS <13 - Normal x-ray but continued concerns regarding c-spine injury - Any focal neurology - A CT head scan is being performed - Initial plain films are abnormal

Immediate CT head (within 1 hour) if:

 GCS < 13 on admission  GCS < 15 2 hours after admission  Suspected open or depressed skull fracture  Suspected skull base fracture (panda eyes, Battle's sign, CSF from nose/ear, bleeding ear)  Focal neurology  Vomiting > 1 episode  Post traumatic seizure  Coagulopathy

Contact neurosurgeon if:

 Persistent GCS < 8 or = 8  Unexplained confusion > 4h  Reduced GCS after admission  Progressive neurological signs  Incomplete recovery post seizure  Penetrating injury  Cerebrospinal fluid leak

Observations

 1/2 hourly GCS until 15

Reference 1. http://guidance.nice.org.uk/CG56/QuickRefGuide/pdf/English 2. Hodgkinson S et al. Early management of head injury: summary of NICE guidance. BMJ 2014 (348):34-37. Next question

A 60-year-old man develops palpitations while on the acute surgical unit. An ECG shows a broad complex tachycardia at a rate of 150 bpm. His blood pressure is 124/82 mmHg and there is no evidence of heart failure. The surgical consultant wants to give rate control (the medical team are not answering their bleeps). Which one of the following is it least appropriate to give?

Procainamide

Lidocaine

Amiodarone

Adenosine

Verapamil

Ventricular tachycardia - verapamil is contraindicated

Verapamil should never be given to a patient with a broad complex tachycardia as it may precipitate ventricular fibrillation in patients with ventricular tachycardia. Adenosine is sometimes given in this situation as a 'trial' if there is a strong suspicion the underlying rhythm is a supraventricular tachycardia with aberrant conduction Please rate this question:

Discuss and give feedback Next question Ventricular tachycardia: management

Whilst a broad complex tachycardia may result from a supraventricular rhythm with aberrant conduction, the European Resuscitation Council advise that in a peri-arrest situation it is assumed to be ventricular in origin

If the patient has adverse signs (systolic BP < 90 mmHg, chest pain, heart failure or rate > 150 beats/min) then immediate cardioversion is indicated. In the absence of such signs antiarrhythmics may be used. If these fail, then electrical cardioversion may be needed with synchronised DC shocks

Drug therapy

 amiodarone: ideally administered through a central line  lidocaine: use with caution in severe left ventricular impairment  procainamide

Verapamil should NOT be used in VT

If drug therapy fails

 electrophysiological study (EPS)  implant able cardioverter-defibrillator (ICD) - this is particularly indicated in patients with significantly impaired LV function

Next question

A 62 year old male attends the hernia clinic. He suddenly develops speech problems, left facial weakness and left sided arm and leg weakness lasting longer than 5 minutes. A CT head shows no intracerebral bleed. What is the next line of management?

Aspirin 300mg

Aspirin 75 mg

Clopidogrel 300mg

Urgent referral for thrombolysis

Carotid endarterectomy

This patient is within 3h of symptom onset of a stroke. Therefore he should be urgently referred to the medical team for thrombolysis, before Aspirin is given. There are concerns that high dose aspirin would increase the risk of intracerebral haemorrhage if thrombolysis is undertaken. This is an example of the type of medical problem you should be aware of as a surgeon, as ultimately you can make a difference by referring QUICKLY to the correct specialty for management. Please rate this question:

Discuss and give feedback Next question Stroke: types

Primary intracerebral  Presents with headache, vomiting, loss of haemorrhage (PICH, c. 10%) consciousness

Total anterior circulation  Involves middle and anterior cerebral arteries infarcts (TACI, c. 15%)  Hemiparesis/hemisensory loss  Homonymous hemianopia  Higher cognitive dysfunction e.g. Dysphasia

Partial anterior circulation  Involves smaller arteries of anterior circulation e.g. infarcts (PACI, c. 25%) upper or lower division of middle cerebral artery  Higher cognitive dysfunction or two of the three TACI features

Lacunar infarcts (LACI, c.  Involves perforating arteries around the internal 25%) capsule, thalamus and basal ganglia  Present with either isolated hemiparesis, hemisensory loss or hemiparesis with limb ataxia

Posterior circulation infarcts  Vertebrobasilar arteries (POCI, c. 25%)  Presents with features of brainstem damage  Ataxia, disorders of gaze and vision, cranial nerve lesions

Lateral medullary syndrome  Wallenberg's syndrome (posterior inferior cerebellar  Ipsilateral: ataxia, nystagmus, dysphagia, facial artery) numbness, cranial nerve palsy  Contralateral: limb sensory loss

Weber's syndrome  Ipsilateral III palsy  Contralateral weakness

------

Anterior cerebral artery

 Contralateral hemiparesis and sensory loss, lower extremity > upper  Disconnection syndrome

Middle cerebral artery

 Contralateral hemiparesis and sensory loss, upper extremity > lower  Contralateral hemianopia  Aphasia (Wernicke's)  Gaze abnormalities

Posterior cerebral artery

 Contralateral hemianopia with macular sparing  Disconnection syndrome

Lacunar

 Present with either isolated hemiparesis, hemisensory loss or hemiparesis with limb ataxia

Lateral medulla (posterior inferior cerebellar artery)

 Ipsilateral: ataxia, nystagmus, dysphagia, facial numbness, cranial nerve palsy e.g.

Horner's

 Contralateral: limb sensory loss

Pontine

 VI nerve: horizontal gaze palsy  VII nerve  Contralateral hemiparesis

Next question

A 45-year-old man is seen in the Emergency Department with nausea, pallor and lethargy. He has no past medical history of note. A cannula is inserted and serum urea and electrolytes show the following

Na+ 140 mmol/l

K+ 6.7 mmol/l

Bicarbonate 14 mmol/l

Urea 18.2 mmol/l

Creatinine 230 µmol/l

What is the most appropriate initial management?

Nebulised salbutamol

Intravenous bicarbonate

Haemodialysis

Insulin/dextrose infusion

Intravenous calcium gluconate

The first priority in this patient is to stabilise the myocardium with intravenous calcium gluconate. Please rate this question:

Discuss and give feedback Next question Management of hyperkalaemia

Untreated hyperkalaemia may cause life-threatening arrhythmias. Precipitating factors should be addressed (e.g. acute renal failure) and aggravating drugs stopped (e.g. ACE inhibitors). Management may be categorised by the aims of treatment

Stabilisation of the cardiac membrane

 Intravenous calcium gluconate

Short-term shift in potassium from extracellular to intracellular fluid compartment

 Combined insulin/dextrose infusion  Nebulised salbutamol

Removal of potassium from the body

 Calcium resonium (orally or enema)  Loop diuretics  Dialysis

Next question

Theme: Visceral injury

A. Ruptured spleen B. Ileum injury C. Duodenal injury D. Urethral injury E. Rectal injury F. Oesophageal injury G. Liver laceration

Please select the most likely injury for the scenario given. Each option may be used once, more than once or not at all.

9. A motorcyclist is involved in a head on road traffic accident with a lorry. He is comatose at scene and trauma series x-rays confirm a pelvic fracture. On rectal examination he has a high riding prostate.

You answered Ruptured spleen

The correct answer is Urethral injury

This is classical for urethral injury. Features of a urethral injury include; pelvic fracture, high riding prostate on digital rectal examination and blood at the urethral meatus. Where this is the suspected diagnosis a suprapubic catheter and urethral contrast studies performed.

10. A cyclist loses control and falls off the side of a road landing on the bicycle handlebars. CT scanning shows a large amount of retroperitoneal air.

You answered Ruptured spleen

The correct answer is Duodenal injury

Theme from 2012 exam Retroperitoneal air is more likely with a duodenal injury. As it is largely retroperitoneal. A handlebar type injury is the commonest cause and the pancreas should be carefully inspected as it too may be injured. It would be unusual for the ileum to be injured in this type of scenario as it is mobile.

11. A 23 year old man is shot in the abdomen. He is haemodynamically stable but on ultrasound he has a large amount of intra abdominal free fluid.

You answered Ruptured spleen

The correct answer is Ileum injury

Small bowel injury is the most common type of injury in this scenario. The enteric contents will tend to result in a large amount of intra abdominal fluid.

Please rate this question:

Discuss and give feedback Next question Trauma management

The cornerstone of trauma management is embodied in the principles of ATLS.

Following trauma there is a trimodal death distribution:

 Immediately following injury. Typically as result of brain or high spinal injuries, cardiac or great vessel damage. Salvage rate is low.  In early hours following injury. In this group deaths are due to phenomena such as splenic rupture, sub dural haematomas and haemopneumothoraces  In the days following injury. Usually due to sepsis or multi organ failure.

Aspects of trauma management

 ABCDE approach.  Tension pneumothoraces will deteriorate with vigorous ventilation attempts.  External haemorrhage is managed as part of the primary survey. As a rule tourniquets should not be used. Blind application of clamps will tend to damage surrounding structures and packing is the preferred method of haemorrhage control.  Urinary catheters and naso gastric tubes may need inserting. Be wary of basal skull fractures and urethral injuries.  Patients with head and neck trauma should be assumed to have a cervical spine injury until proven otherwise.

Thoracic injuries

 Simple pneumothorax  Mediastinal traversing wounds  Tracheobronchial tree injury  Haemothorax  Blunt cardiac injury  Diaphragmatic injury  Aortic disruption  Pulmonary contusion

Management of thoracic trauma

 Simple pneumothorax insert chest drain. Aspiration is risky in trauma as pneumothorax may be from lung laceration and convert to tension pneumothorax.  Mediastinal traversing wounds These result from situations like stabbings. Exit and entry wounds in separate hemithoraces. The presence of a mediastinal haematoma indicates the likelihood of a great vessel injury. All patients should undergo CT angiogram and oesophageal contrast swallow. Indications for thoracotomy are largely related to blood loss and will be addressed below.  Tracheobronchial tree injury Unusual injuries. In blunt trauma most injuries occur within 4cm of the carina. Features suggesting this injury include haemoptysis and surgical emphysema. These injuries have a very large air leak and may have tension pneumothorax.  Haemothorax Usually caused by laceration of lung vessel or internal mammary artery by rib fracture. Patients should all have a wide bore 36F chest drain. Indications for thoracotomy include loss of more than 1.5L blood initially or ongoing losses of >200ml per hour for >2 hours.  Cardiac contusions Usually cardiac arrhythmias, often overlying sternal fracture. Perform echocardiography to exclude pericardial effusions and tamponade. Risk of arrhythmias falls after 24 hours.  Diaphragmatic injury Usually left sided. Direct surgical repair is performed.  Traumatic aortic disruption Commonest cause of death after RTA or falls. Usually incomplete laceration near ligamentum arteriosum. All survivors will have contained haematoma. Only 1-2% of patients with this injury will have a normal chest x-ray.  Pulmonary contusion Common and lethal. Insidious onset. Early intubation and ventilation.

Abdominal trauma

 Deceleration injuries are common.  In blunt trauma requiring laparotomy the spleen is most commonly injured (40%)  Stab wounds traverse structures most commonly liver (40%)  Gunshot wounds have variable effects depending upon bullet type. Small bowel is most commonly injured (50%)  Patients with stab wounds and no peritoneal signs up to 25% will not enter the peritoneal cavity  Blood at urethral meatus suggests a urethral tear  High riding prostate on PR = urethral disruption  Mechanical testing for pelvic stability should only be performed once

Investigations in abdominal trauma

Diagnostic Peritoneal Abdominal CT scan USS Lavage

Indication Document bleeding if Document organ injury if Document fluid if hypotensive normotensive hypotensive

Advantages Early diagnosis and Most specific for Early diagnosis, non sensitive; 98% accurate localising injury; 92 to invasive and repeatable; 98% accurate 86 to 95% accurate

Disadvantages Invasive and may miss Location of scanner away Operator dependent and retroperitoneal and from facilities, time taken may miss diaphragmatic injury for reporting, need for retroperitoneal injury contrast

 Amylase may be normal following pancreatic trauma  Urethrography if suspected urethral injury

Next question

A 22 year old man has a full thickness burn of his leg after being trapped in a burning car. There are no fractures of the limb. There burn is well circumscribed. After 2 hours he complains of tingling of his leg and it appears dusky. What is the best management for this?

Fasciotomy

Escharotomy

Angioplasty

Pain control

Anticoagulation

The full thickness burn has oedema which is affecting the peripheral circulation. Therefore the burn needs to be divided (not the fascia) to allow normal circulation to return. Please rate this question:

Discuss and give feedback Next question Burns

Burns may be thermal, chemical or electrical. In the former category are burns which occur as a result of heat. Chemical burns occur when the skin is exposed to an extremely caustic or alkaline substance. Electrical burns occur following exposure to electrical current. The immediate management includes removal of the burning source which usually includes irrigation of the burned area. A detailed assessment then needs to be made of the extent of the burns and a number of charts are available for recording this information. The degree of injury relates to the temperature and duration of exposure. Most domestic burns are mainly scalds in young children.

Following the burn, there is a local response with progressive tissue loss and release of inflammatory cytokines. Systemically, there are cardiovascular effects resulting from fluid loss and sequestration of fluid into the third space. There is a marked catabolic response. Immunosupression is common with large burns and bacterial translocation from the gut lumen is a recognised event. Sepsis is a common cause of death following major burns.

Types of burn

Type of burn Skin layers Skin Blanching Management affected appearance

Epidermal/Superficial Epidermis Red, moist Yes

Superficial partial Epidermis and part Pale, dry Yes Normally heals thickness of papillary dermis with no affected intervention

Deep partial thickness Epidermis, whole Mottled red No Needs surgical papillary dermis colour intervention affected (depending on site)

Full thickness Whole skin layer Dry, leathery No Burns centre and subcutaneous hard wound tissue affected

Depth of burn assessment

 Bleeding on needle prick  Sensation  Appearance  Blanching to pressure

Percentage burn estimation Lund Browder chart: most accurate even in children Wallace rule of nines Palmar surface: surface area palm = 0.8% burn

>15% body surface area burns in adults needs urgent burn fluid resuscitation

Transfer to burn centre if:

 Need burn shock resuscitation  Face/hands/genitals affected  Deep partial thickness or full thickness burns  Significant electrical/chemical burns

Management The initial aim is to stop the burning process and resuscitate the patient. Intravenous fluids will be required for children with burns greater than 10% of total body surface area. Adults with burns greater than 15% of total body surface area will also require IV fluids. The fluids are calculated using the Parkland formula which is; volume of fluid= total body surface area of the burn % x weight (Kg) x4. Half of the fluid is administered in the first 8 hours. A urinary catheter should be inserted. Analgesia should be given. Complex burns, burns involving the hand perineum and face and burns >10% in adults and >5% in children should be transferred to a burns unit.

Circumferential burns affecting a limb or severe torso burns impeding respiration may require escharotomy to divide the burnt tissue.

Conservative management is appropriate for superficial burns and mixed superficial burns that will heal in 2 weeks. More complex burns may require excision and skin grafting. Excision and primary closure is not generally practised as there is a high risk of infection.

There is no evidence to support the use of anti microbial prophylaxis or topical antibiotics in burn patients.

Escharotomies

 Indicated in circumferential full thickness burns to the torso or limbs.  Careful division of the encasing band of burn tissue will potentially improve ventilation (if the burn involves the torso), or relieve compartment syndrome and oedema (where a limb is involved)

References www.euroburn.org/e107files/downloads/guidelinesburncare.pdf

Barajas-Nava LA, López-Alcalde J, Roqué i Figuls M, Solà I, Bonfill Cosp X. Antibiotic prophylaxis for preventing burn wound infection. Cochrane Database of Systematic Reviews 2013, Issue 6. Art. No.: CD008738. DOI: 10.1002/14651858.CD008738.pub2.

Hettiaratchy S & Papini R. Initial management of a major burn: assessment and resuscitation. BMJ 2004;329:101-103 Next question

A 28 year old man is in the surgical intensive care unit. He has suffered a flail chest injury several hours earlier and he was intubated and ventilated. Over the past few minutes he has become increasingly hypoxic and is now needing increased ventilation pressures. What is the most common cause?

Pulmonary embolism

Cardiac tamponade

Fat embolism

Tension pneumothorax

Adult respiratory distress syndrome

Theme from April 2011 Exam

A flail chest segment may lacerate the underlying lung and create a flap valve. A tension pneumothorax can be created by intubation and ventilation in this situation. Sudden hypoxia and increased ventilation pressure are clues. Please rate this question:

Discuss and give feedback Next question Tension Pneumothorax

Tension pneumothorax is a state of positive pressure within a pneumothorax throughout the respiratory cycle. A breach in the pleura allows air into the intra pleural space via a one way valve. The initial pressure pneumothorax expands until positive pressure is present throughout the respiratory cycle. The risk is greatest in the ventilated trauma patient as positive pressure is used. Undiagnosed tension pneumothorax accounts for 3.8% of trauma deaths.

Clinically, the classic features include chest pain, dyspnoea, hypoxia, hypotension, tracheal deviation, ipsilateral hyperpercussion note, decreased air entry. In ventilated patients, cardiovascular disturbance and sub cutaneous emphysema are relatively common and more so than in a case where the patient is breathing spontaneously.

Chest x-ray features

 Lung collapse towards the hilum  Diaphragmatic depression Increased rib separation  Increased thoracic volume  Ipsilateral flattening of the heart border  Contra lateral mediastinal deviation

Management

Immediate needle decompression followed by definitive wide bore chest drain insertion

Image showing mediastinal shift with a tension pneumothorax

Image sourced from Wikipedia

Next question

A 10 year old boy is playing with a firework which explodes and he sustains a full thickness burn to his left arm. Which of the following statements is not characteristic of this situation?

They have a leathery appearance

The burn area is extremely painful until skin grafted

They always heal with scarring

Blanching does not occur under pressure

Absence of,or few, blisters

Full thickness burns involve complete injury to the dermis and sub dermal appendages. They have a leathery, often white appearance. They are initially insensate although pain often occurs during healing following skin grafting. They do not blanch under pressure. Please rate this question:

Discuss and give feedback Next question Burns

Burns may be thermal, chemical or electrical. In the former category are burns which occur as a result of heat. Chemical burns occur when the skin is exposed to an extremely caustic or alkaline substance. Electrical burns occur following exposure to electrical current. The immediate management includes removal of the burning source which usually includes irrigation of the burned area. A detailed assessment then needs to be made of the extent of the burns and a number of charts are available for recording this information. The degree of injury relates to the temperature and duration of exposure. Most domestic burns are mainly scalds in young children.

Following the burn, there is a local response with progressive tissue loss and release of inflammatory cytokines. Systemically, there are cardiovascular effects resulting from fluid loss and sequestration of fluid into the third space. There is a marked catabolic response. Immunosupression is common with large burns and bacterial translocation from the gut lumen is a recognised event. Sepsis is a common cause of death following major burns.

Types of burn

Type of burn Skin layers Skin Blanching Management affected appearance

Epidermal/Superficial Epidermis Red, moist Yes

Superficial partial Epidermis and part Pale, dry Yes Normally heals thickness of papillary dermis with no affected intervention

Deep partial thickness Epidermis, whole Mottled red No Needs surgical papillary dermis colour intervention affected (depending on site)

Full thickness Whole skin layer Dry, leathery No Burns centre and subcutaneous hard wound tissue affected

Depth of burn assessment

 Bleeding on needle prick  Sensation  Appearance  Blanching to pressure

Percentage burn estimation Lund Browder chart: most accurate even in children Wallace rule of nines Palmar surface: surface area palm = 0.8% burn

>15% body surface area burns in adults needs urgent burn fluid resuscitation

Transfer to burn centre if:

 Need burn shock resuscitation  Face/hands/genitals affected  Deep partial thickness or full thickness burns  Significant electrical/chemical burns

Management The initial aim is to stop the burning process and resuscitate the patient. Intravenous fluids will be required for children with burns greater than 10% of total body surface area. Adults with burns greater than 15% of total body surface area will also require IV fluids. The fluids are calculated using the Parkland formula which is; volume of fluid= total body surface area of the burn % x weight (Kg) x4. Half of the fluid is administered in the first 8 hours. A urinary catheter should be inserted. Analgesia should be given. Complex burns, burns involving the hand perineum and face and burns >10% in adults and >5% in children should be transferred to a burns unit.

Circumferential burns affecting a limb or severe torso burns impeding respiration may require escharotomy to divide the burnt tissue.

Conservative management is appropriate for superficial burns and mixed superficial burns that will heal in 2 weeks. More complex burns may require excision and skin grafting. Excision and primary closure is not generally practised as there is a high risk of infection.

There is no evidence to support the use of anti microbial prophylaxis or topical antibiotics in burn patients.

Escharotomies

 Indicated in circumferential full thickness burns to the torso or limbs.  Careful division of the encasing band of burn tissue will potentially improve ventilation (if the burn involves the torso), or relieve compartment syndrome and oedema (where a limb is involved)

References www.euroburn.org/e107files/downloads/guidelinesburncare.pdf

Barajas-Nava LA, López-Alcalde J, Roqué i Figuls M, Solà I, Bonfill Cosp X. Antibiotic prophylaxis for preventing burn wound infection. Cochrane Database of Systematic Reviews 2013, Issue 6. Art. No.: CD008738. DOI: 10.1002/14651858.CD008738.pub2.

Hettiaratchy S & Papini R. Initial management of a major burn: assessment and resuscitation. BMJ 2004;329:101-103 Next question

Theme: Thoracic injuries

A. Pneumothorax B. Tension pneumothorax C. Flail chest D. Cardiac tamponade E. Aorta rupture F. Cardiac contusion G. H. Acute phrenic nerve injury

For each of the scenarios given, please select the most likely underlying injury. Each option may be used once, more than once or not at all.

15. An 18 year old student is involved in a car crash, with another car crashing into the side of the car. A CXR shows an indistinct left hemidiaphragm.

You answered Pneumothorax

The correct answer is Diaphragmatic rupture

A lateral blunt injury during a road traffic accident is a common cause of diaphragmatic rupture. Diagnosis is usually evident on chest x-ray. CXR changes include non visible diaphragm, bowel loops in the hemithorax and displacement of the mediastinum. In most cases direct surgical repair is the best option.

16. A 19 year old motorcyclist is involved in a road traffic accident. His chest movements are irregular. He is found to have multiple rib fractures, with 2 fractures in the 3rd rib and 3 fractures in the 4th rib.

You answered Pneumothorax

The correct answer is Flail chest

Multiple rib fractures with > or = 2 rib fractures in more than 2 ribs is diagnosed as a flail chest. This is associated with pulmonary contusion.

17. A 19 year old student falls from a 2nd floor window. He is persistently hypotensive. A CXR shows depression of the left main bronchus and deviation of the trachea to the right.

You answered Pneumothorax

The correct answer is Aorta rupture

He has a deceleration injury, with persistent hypotension (contained haematoma). This should indicate aorta rupture. Widened mediastinum may not always be present on a CXR. A CT angiogram will provide clearer evidence of the extent of injury. The presence of persistent hypotension, from a early stage is more consistent with haematoma than a tension pneumothorax in which it occurs as a final periarrest phenomena. CXR findings in diaphragmatic rupture: Hemidiaphragm is not visible Bowel loops in the lower half of the hemi-thorax Mediastinum is displaced

Please rate this question:

Discuss and give feedback Next question Thoracic trauma

Types of thoracic trauma

Tension  Often laceration to lung parenchyma with flap pneumothorax  Pressure develops in thorax  Most common cause is mechanical ventilation in patient with pleural injury  Symptoms overlap with cardiac tamponade, hyper-resonant percussion note is more likely in tension pnemothorax

Flail chest  Chest wall disconnects from thoracic cage  Multiple rib fractures (at least two fractures per rib in at least two ribs)  Associated with pulmonary contusion  Abnormal chest motion  Avoid over hydration and fluid overload

Pneumothorax  Most common cause is lung laceration with air leakage  Most traumatic pneumothoraces should have a chest drain  Patients with traumatic pneumothorax should never be mechanically ventilated until a chest drain is inserted

Haemothorax  Most commonly due to laceration of lung, intercostal vessel or internal mammary artery  Haemothoraces large enough to appear on CXR are treated with large bore chest drain  Surgical exploration is warranted if >1500ml blood drained immediately

Cardiac tamponade  Beck's triad: elevated venous pressure, reduced arterial pressure, reduced heart sounds  Pulsus paradoxus  May occur with as little as 100ml blood

Pulmonary contusion  Most common potentially lethal chest injury  Arterial blood gases and pulse oximetry important  Early intubation within an hour if significant hypoxia

Blunt cardiac injury  Usually occurs secondary to chest wall injury  ECG may show features of myocardial infarction  Sequelae: hypotension, arrhythmias, cardiac wall motion abnormalities

Aorta disruption  Deceleration injuries  Contained haematoma  Widened mediastinum

Diaphragm  Most due to motor vehicle accidents and blunt trauma causing disruption large radial tears (laceration injuries result in small tears)  More common on left side  Insert gastric tube, may pass into intrathoracic stomach

Mediastinal  Entrance wound in one hemithorax and exit wound/foreign body traversing wounds in opposite hemithorax  Mediastinal haematoma or pleural cap suggests great vessel injury  Mortality is 20%

References ATLS Manual 8th Edition Next question Which of the features below, following a head injury, is not an indication for an immediate CT head scan in children?

Drowsiness

A single, discrete episode of vomiting

A 9 month old child with a 6cm haematoma on the head

Numb left arm

Suspicion of a non accidental head injury

Whilst not an indication for immediate CT there should be a low threshold for admission and observation.

Please rate this question:

Discuss and give feedback

Next question

Head injury paediatrics

Criteria for immediate request for CT scan of the head (children) * Loss of consciousness lasting more than 5 minutes (witnessed) * Amnesia (antegrade or retrograde) lasting more than 5 minutes * Abnormal drowsiness * Three or more discrete episodes of vomiting * Clinical suspicion of non-accidental injury * Post-traumatic seizure but no history of epilepsy * GCS less than 14, or for a baby under 1 year GCS (paediatric) less than 15, on assessment in the emergency department * Suspicion of open or depressed skull injury or tense fontanelle * Any sign of basal skull fracture (haemotympanum, panda' eyes, cerebrospinal fluid leakage from the ear or nose, Battle's sign) * Focal neurological deficit * If under 1 year, presence of , swelling or laceration of more than 5 cm on the head * Dangerous mechanism of injury (high-speed road traffic accident either as pedestrian, cyclist or vehicle occupant, fall from a height of greater than 3 m, high-speed injury from a projectile or an object)

Next question

Theme: Management of osteomyelitis

A. Lautenbach regime B. Below knee amputation C. Hindquater amputation D. Above knee amputation E. Removal of metalwork and implantation of local antibiotics F. Removal of metalwork and bone grafting G. Intravenous antibiotics

Which option is the best management plan? Each option may be used once, more than once or not at all

19. A 65 year old type 2 diabetic with poor glycaemic control is admitted with forefoot cellulitis. X-ray of the foot shows some evidence of osteomyelitis of the 2nd ray but overlying skin is healthy.

You answered Lautenbach regime

The correct answer is Intravenous antibiotics

It is worth attempting to try and resolve this situation with antibiotics at first presentation. A primary amputation will not heal well and may result in progressive surgery.

20. A 28 year old infantryman is shot in the leg during combat. Primary debridement and lavage of the wound is undertaken. Several months post surgery there is ongoing discharge from a sinus originating in the proximal femur, X-ray and MRI shows evidence of osteomyelitis of the proximal femur. There are no obvious sequestra.

Lautenbach regime

This involves local administration of antibiotics via intramedullary lines and is an intensive regime. However, the morbidity of a high above knee or hindquater amputation makes conservative management an attractive option.

21. A 70 year old man undergoes a revision total hip replacement. 10 days post operatively the hip dislocates and pus is discharging from the wound. He is systemically unwell with a temperature of 38.5 and WCC 19.

You answered Lautenbach regime

The correct answer is Removal of metalwork and implantation of local antibiotics

Removal of metal work implantation of gentamicin beads and delayed revision is the mainstay of managing this complication.

Please rate this question:

Discuss and give feedback Next question Osteomyelitis

Infection of the bone

Causes

 S aureus and occasionally Enterobacter or Streptococcus species  In sickle cell: Salmonella species

Clinical features

 Erythema  Pain  Fever

Investigation

 X-ray: lytic centre with a ring of sclerosis  Bone biopsy and culture

Treatment

 Prolonged antibiotics  Sequestra may need surgical removal

Next question

What is the least likely examination finding in patients with Le Fort II fractures?

Excessive mobility of the palate

Paraesthesia in the region supplied by the inferior alveolar nerve

Malocclusion of the teeth

Enopthalmos

Parasthesia in the region supplied by the infraorbital nerve

Le Fort II fractures have a pyramidal shape. The fracture line involves the orbit and extends to involve the bridge of the nose and the ethmoids. In continues to involve the infraorbital rim and usually through the infraorbital foramen. As a result infraorbital parasthesia, palatal mobility and malocclusion are common findings. Severe fractures may result in endopthalmos. However, the fracture does not, by definition, involve the inferior alveolar nerve. Please rate this question:

Discuss and give feedback Next question Craniomaxillofacial injuries

Craniomaxillofacial injuries in the UK are due to:

 Interpersonal violence (52%)  Motor vehicle accidents (16%)  Sporting injuries (19%)  Falls (11%)

Le Fort Fractures Grade Feature

Le The fracture extends from the nasal septum to the lateral pyriform rims, travels Fort 1 horizontally above the teeth apices, crosses below the zygomaticomaxillary junction, and Grade Feature

traverses the pterygomaxillary junction to interrupt the pterygoid plates.

Le These fractures have a pyramidal shape and extend from the nasal bridge at or below the Fort 2 nasofrontal suture through the frontal process of the maxilla, inferolaterally through the lacrimal bones and inferior orbital floor and rim through or near the inferior orbital foramen, and inferiorly through the anterior wall of the maxillary sinus; it then travels under the zygoma, across the pterygomaxillary fissure, and through the pterygoid plates.

Le These fractures start at the nasofrontal and frontomaxillary sutures and extend posteriorly Fort 3 along the medial wall of the orbit through the nasolacrimal groove and ethmoid bones. The thicker sphenoid bone posteriorly usually prevents continuation of the fracture into the optic canal. Instead, the fracture continues along the floor of the orbit along the inferior orbital fissure and continues superolaterally through the lateral orbital wall, through the zygomaticofrontal junction and the zygomatic arch. Intranasally, a branch of the fracture extends through the base of the perpendicular plate of the ethmoid, through the vomer, and through the interface of the pterygoid plates to the base of the sphenoid. This type of fracture predisposes the patient to CSF rhinorrhea more commonly than the other types.

Ocular injuries Superior orbital fissure syndrome Severe force to the lateral wall of the orbit resulting in compression of neurovascular structures. Results in :

 Complete opthalmoplegia and ptosis (Cranial nerves 3, 4, 6 and nerve to levator palpebrae superioris)  Relative afferent pupillary defect  Dilatation of the pupil and loss of accommodation and corneal reflexes  Altered sensation from forehead to vertex (frontal branch of trigeminal nerve)

Orbital blow out fracture Typically occurs when an object of slightly larger diameter than the orbital rim strikes the incompressible eyeball. The bone fragment is displaced downwards into the antral cavity, remaining attached to the orbital periosteum. Periorbital fat may be herniated through the defect, interfering with the inferior rectus and inferior oblique muscles which are contained within the same fascial sheath. This prevents upward movement and outward rotation of the eye and the patient experiences diplopia on upward gaze. The initial bruising and swelling may make assessment difficult and patients should usually be reviewed 5 days later. Residual defects may require orbital floor reconstruction.

Nasal Fractures

 Common injury  Ensure new and not old deformity  Control epistaxis  CSF rhinorrhoea implies that the cribriform plate has been breached and antibiotics will be required.  Usually best to allow bruising and swelling to settle and then review patient clinically. Major persistent deformity requires fracture manipulation, best performed within 10 days of injury.

Retrobulbar haemorrhage Rare but important ocular emergency. Presents with:

 Pain (usually sharp and within the globe)  Proptosis  Pupil reactions are lost  Paralysis (eye movements lost)  Visual acuity is lost (colour vision is lost first)

May be the result of Le Fort type facial fractures.

Management:

 Mannitol 1g/Kg as 20% infusion, Osmotic diuretic, Contra-indicated in congestive heart failure and pulmonary oedema  Acetazolamide 500mg IV, (Monitor FBC/U+E) Reduces aqueous pressure by inhibition of carbonic anhydrase (used in glaucoma)  Dexamethasone 8mg orally or intravenously  In a traumatic setting an urgent cantholysis may be needed prior to definitive surgery.

Consider Papaverine 40mg smooth muscle relaxant Dextran 40 500mls IV improves perfusion Next question

A Medical F1 phones you as he is concerned his patient has had a major internal bleed. The patient is 42 years old and is known to have sickle cell anaemia. His blood results are:

Hb 3.7 g /dl Reticulocyte count 0.4%

His Hb is normally 7g/dl. What is the diagnosis?

Psoas haemorrhage

Acute sequestration

Parvovirus

Splenic haemorrhage

Acute haemolysis

A sudden anemia and a LOW reticulocute count indicates parvovirus. Acute sequestration and haemolysis causes a high reticulocyte count. There is no clinical indication to suspect a bleed, therefore you can advise the F1 not to panic and to speak to the haematologists! Please rate this question:

Discuss and give feedback Next question Sickle cell anaemia

 Autosomal recessive  Single base mutation  Deoxygenated cells become sickle in shape  Causes: short red cell survival, obstruction of microvessels and infarction  Sickling is precipitated by: dehydration, infection, hypoxia  Manifest at 6 months age  Africans, Middle East, Indian  Diagnosis: Hb electrophoresis

Sickle crises

 Bone pain  Pleuritic chest pain: acute sickle chest syndrome commonest cause of death  CVA, seizures  Papillary necrosis  Splenic infarcts  Priapism  Hepatic pain

Hb does not fall during a crisis, unless there is

 Aplasia: parvovirus  Acute sequestration  Haemolysis

Long-term complications

 Infections: Streptococcus pnemoniae  Chronic leg ulcers  Gallstones: haemolysis  Aseptic necrosis of bone  Chronic renal disease  Retinal detachment, proliferative retinopathy

Surgical complications

 Bowel ischaemia  Cholecystitis  Avascular necrosis

Management

 Supportive  Hydroxyurea  Repeated transfusions pre operatively  Exchange transfusion in emergencies

Sickle cell trait

 Heterozygous state  Asymptomatic  Symptoms associated with extreme situations ie anaesthesia complications  Protective against Plasmodium falciparum

Next question

Theme: Management of chest trauma

A. Thoracotomy in operating theatre B. 36F intercostal chest drain C. 14F intercostal chest drain D. Active observation E. Thoracotomy in the emergency room F. MRI of aortic arch G. Bronchoscopy H. Pericardiocentesis I. Further transfusion

For each of the following scenarios please select the most appropriate next stage of management. Each option may be used once, more than once or not at all.

24. A 30 year old male is stabbed outside a nightclub he has a brisk haemoptysis and in casualty has a chest drain inserted into the left chest. This drained 750ml frank blood. He fails to improve with this intervention. He has received 4 units of blood. His CVP is now 13.

You answered Thoracotomy in operating theatre

The correct answer is Pericardiocentesis

This man has cardiac tamponade. The raised CVP in the setting of haemodynamic compromise is the pointer to this. Whilst he will almost certainly require surgery, he requires urgent decompression of his heart first.

25. A 26 year old male falls from a cliff. He suffers from multiple fractures and has a right sided pneumothorax that has collapsed a 1/3 of his lung. He has no respiratory compromise.

You answered Thoracotomy in operating theatre

The correct answer is 14F intercostal chest drain

Simple observation is unsafe as he will almost certainly have suffered an oblique laceration to his lung. These can become tension pneumothoraces. In the absence of blood a 36 F drain is probably not required

26. An 18 year old male is shot in the left chest he was unstable but his blood pressure has improved with 1 litre of colloid. His chest x-ray shows a left sided pneumothorax with no lung visible.

You answered Thoracotomy in operating theatre

The correct answer is 36F intercostal chest drain

This man requires wide bore intercostal tube drainage. Smaller intercostal chest drains can become occluded with blood clot and fail to function adequately.

Please rate this question:

Discuss and give feedback Next question Thoracic trauma

Types of thoracic trauma

Tension  Often laceration to lung parenchyma with flap pneumothorax  Pressure develops in thorax  Most common cause is mechanical ventilation in patient with pleural injury  Symptoms overlap with cardiac tamponade, hyper-resonant percussion note is more likely in tension pnemothorax

Flail chest  Chest wall disconnects from thoracic cage  Multiple rib fractures (at least two fractures per rib in at least two ribs)  Associated with pulmonary contusion  Abnormal chest motion  Avoid over hydration and fluid overload

Pneumothorax  Most common cause is lung laceration with air leakage  Most traumatic pneumothoraces should have a chest drain  Patients with traumatic pneumothorax should never be mechanically ventilated until a chest drain is inserted

Haemothorax  Most commonly due to laceration of lung, intercostal vessel or internal mammary artery  Haemothoraces large enough to appear on CXR are treated with large bore chest drain  Surgical exploration is warranted if >1500ml blood drained immediately

Cardiac tamponade  Beck's triad: elevated venous pressure, reduced arterial pressure, reduced heart sounds  Pulsus paradoxus  May occur with as little as 100ml blood

Pulmonary contusion  Most common potentially lethal chest injury  Arterial blood gases and pulse oximetry important  Early intubation within an hour if significant hypoxia

Blunt cardiac injury  Usually occurs secondary to chest wall injury  ECG may show features of myocardial infarction  Sequelae: hypotension, arrhythmias, cardiac wall motion abnormalities

Aorta disruption  Deceleration injuries  Contained haematoma  Widened mediastinum

Diaphragm  Most due to motor vehicle accidents and blunt trauma causing disruption large radial tears (laceration injuries result in small tears)  More common on left side  Insert gastric tube, may pass into intrathoracic stomach

Mediastinal  Entrance wound in one hemithorax and exit wound/foreign body traversing wounds in opposite hemithorax  Mediastinal haematoma or pleural cap suggests great vessel injury  Mortality is 20%

References ATLS Manual 8th Edition Next question

Theme: Management of burns

A. Escharotomy B. Endotracheal intubation C. Broad spectrum intravenous antibiotics D. Intravenous fluids calculated according to extent of burned area E. Discharge with review in outpatients F. Transfer to regional burn centre once stabilised G. Split thickness skin graft H. Full thickness skin graft

What is the best management for the scenario given? Each option may be used once, more than once or not at all.

27. A 34 year old women trips over and falls into a bonfire whilst intoxicated at a party. She suffers burns to her arms, torso and face. These are calculated to be 25% body surface area. She is otherwise stable. The burns to the torso are superficial, her left forearm has a full thickness burn and the burns to her face are superficial. There is no airway compromise. She has received 1000ml of intravenous Hartman's solution, with a further 1000ml prescribed to run over 4 hours.

You answered Escharotomy

The correct answer is Transfer to regional burn centre once stabilised

This women has been resuscitated and requires transfer for specialist management.

28. A 20 year old man is trapped in a warehouse fire. He has sustained 60% burns to his torso and limbs. The limb burns are partial thickness but the torso burns are full thickness. He was intubated by paramedics at the scene and is receiving intravenous fluids. His ventilation pressure requirements are rising.

Escharotomy

He requires an escharotomy as this will be contributing to impaired ventilation.

29. An 18 year old man accidentally pours boiling water onto his left arm. The area is erythematous and has a measuring 5cm. The wound is extremely painful.

You answered Escharotomy

The correct answer is Discharge with review in outpatients

This is a superficial burn and should recover with no further input than simple dressings, an alternative would be deroofing the blister and applying dressings prior to outpatient review.

Please rate this question:

Discuss and give feedback Next question Burns

Burns may be thermal, chemical or electrical. In the former category are burns which occur as a result of heat. Chemical burns occur when the skin is exposed to an extremely caustic or alkaline substance. Electrical burns occur following exposure to electrical current. The immediate management includes removal of the burning source which usually includes irrigation of the burned area. A detailed assessment then needs to be made of the extent of the burns and a number of charts are available for recording this information. The degree of injury relates to the temperature and duration of exposure. Most domestic burns are mainly scalds in young children.

Following the burn, there is a local response with progressive tissue loss and release of inflammatory cytokines. Systemically, there are cardiovascular effects resulting from fluid loss and sequestration of fluid into the third space. There is a marked catabolic response. Immunosupression is common with large burns and bacterial translocation from the gut lumen is a recognised event. Sepsis is a common cause of death following major burns.

Types of burn

Type of burn Skin layers Skin Blanching Management affected appearance

Epidermal/Superficial Epidermis Red, moist Yes

Superficial partial Epidermis and part Pale, dry Yes Normally heals thickness of papillary dermis with no affected intervention

Deep partial thickness Epidermis, whole Mottled red No Needs surgical papillary dermis colour intervention affected (depending on site) Full thickness Whole skin layer Dry, leathery No Burns centre and subcutaneous hard wound tissue affected

Depth of burn assessment

 Bleeding on needle prick  Sensation  Appearance  Blanching to pressure

Percentage burn estimation Lund Browder chart: most accurate even in children Wallace rule of nines Palmar surface: surface area palm = 0.8% burn

>15% body surface area burns in adults needs urgent burn fluid resuscitation

Transfer to burn centre if:

 Need burn shock resuscitation  Face/hands/genitals affected  Deep partial thickness or full thickness burns  Significant electrical/chemical burns

Management The initial aim is to stop the burning process and resuscitate the patient. Intravenous fluids will be required for children with burns greater than 10% of total body surface area. Adults with burns greater than 15% of total body surface area will also require IV fluids. The fluids are calculated using the Parkland formula which is; volume of fluid= total body surface area of the burn % x weight (Kg) x4. Half of the fluid is administered in the first 8 hours. A urinary catheter should be inserted. Analgesia should be given. Complex burns, burns involving the hand perineum and face and burns >10% in adults and >5% in children should be transferred to a burns unit.

Circumferential burns affecting a limb or severe torso burns impeding respiration may require escharotomy to divide the burnt tissue.

Conservative management is appropriate for superficial burns and mixed superficial burns that will heal in 2 weeks. More complex burns may require excision and skin grafting. Excision and primary closure is not generally practised as there is a high risk of infection.

There is no evidence to support the use of anti microbial prophylaxis or topical antibiotics in burn patients.

Escharotomies

 Indicated in circumferential full thickness burns to the torso or limbs.  Careful division of the encasing band of burn tissue will potentially improve ventilation (if the burn involves the torso), or relieve compartment syndrome and oedema (where a limb is involved)

References www.euroburn.org/e107files/downloads/guidelinesburncare.pdf

Barajas-Nava LA, López-Alcalde J, Roqué i Figuls M, Solà I, Bonfill Cosp X. Antibiotic prophylaxis for preventing burn wound infection. Cochrane Database of Systematic Reviews 2013, Issue 6. Art. No.: CD008738. DOI: 10.1002/14651858.CD008738.pub2.

Hettiaratchy S & Papini R. Initial management of a major burn: assessment and resuscitation. BMJ 2004;329:101-103 Next question

A 52 year old male type 2 diabetic is admitted to the vascular ward for a femoral popliteal bypass. He suddenly develops expressive dysphasia and marked right sided weakness. The Senior house officer arranges a CT head scan which shows a 60% left middle cerebral artery territory infarct. There are no beds on the stroke unit. Overnight the patient becomes unresponsive and a CT head confirms no bleed. What is the next best management option?

IV heparin

Clopidogrel

Burr hole surgery

Aspirin

Hemicranieotomy

The likely cause for the reduced consciousness is raised intracranial pressure due to increasing cerebral oedema related to the infarct. In this situation, urgent neurosurgical review is needed for possible decompressive hemicranieotomy to relieve the pressure. Ideally no further antiplatelet or anticoagulation therapy should be given until a plan for surgery is confirmed.

Indications for hemicranieotomy include:

 Age under 60 years  Clinical deficit in middle cerebral artery territory  Decreased consciousness  >50% territory infarct

Please rate this question:

Discuss and give feedback Next question Stroke: types

Primary intracerebral  Presents with headache, vomiting, loss of haemorrhage (PICH, c. 10%) consciousness Total anterior circulation  Involves middle and anterior cerebral arteries infarcts (TACI, c. 15%)  Hemiparesis/hemisensory loss  Homonymous hemianopia  Higher cognitive dysfunction e.g. Dysphasia

Partial anterior circulation  Involves smaller arteries of anterior circulation e.g. infarcts (PACI, c. 25%) upper or lower division of middle cerebral artery  Higher cognitive dysfunction or two of the three TACI features

Lacunar infarcts (LACI, c.  Involves perforating arteries around the internal 25%) capsule, thalamus and basal ganglia  Present with either isolated hemiparesis, hemisensory loss or hemiparesis with limb ataxia

Posterior circulation infarcts  Vertebrobasilar arteries (POCI, c. 25%)  Presents with features of brainstem damage  Ataxia, disorders of gaze and vision, cranial nerve lesions

Lateral medullary syndrome  Wallenberg's syndrome (posterior inferior cerebellar  Ipsilateral: ataxia, nystagmus, dysphagia, facial artery) numbness, cranial nerve palsy  Contralateral: limb sensory loss

Weber's syndrome  Ipsilateral III palsy  Contralateral weakness

------

Anterior cerebral artery

 Contralateral hemiparesis and sensory loss, lower extremity > upper  Disconnection syndrome

Middle cerebral artery

 Contralateral hemiparesis and sensory loss, upper extremity > lower  Contralateral hemianopia  Aphasia (Wernicke's)  Gaze abnormalities

Posterior cerebral artery

 Contralateral hemianopia with macular sparing  Disconnection syndrome

Lacunar

 Present with either isolated hemiparesis, hemisensory loss or hemiparesis with limb ataxia

Lateral medulla (posterior inferior cerebellar artery)

 Ipsilateral: ataxia, nystagmus, dysphagia, facial numbness, cranial nerve palsy e.g.

Horner's

 Contralateral: limb sensory loss

Pontine

 VI nerve: horizontal gaze palsy  VII nerve  Contralateral hemiparesis

Next question

A 56-year-old female is admitted to ITU with a severe pancreatitis. Thyroid function tests show:

TSH = 0.5 Low Thyroxine = 1.0 Low T3 = 0.5 Low

What is the most likely cause?

Sick euthyroid syndrome

Graves disease

Hashimotos thyroiditis

Levothyroxine

None of the above

This patient has sick euthyroid syndrome as all thyroid parameters are reduced. Graves disease and levothyroxine will cause hyperthyroidism (low TSH and elevated thyroxine/T3). Hashimotos thyroiditis is associated with hypothyroidism (high TSH and low thyroxine/T3).

Please rate this question:

Discuss and give feedback

Sick euthyroid syndrome

In sick euthyroid syndrome (now referred to as non-thyroidal illness) it is often said that everything (TSH, thyroxine and T3) is low. In the majority of cases however the TSH level is within the normal range (inappropriately normal given the low thyroxine and T3).

Changes are reversible upon recovery from the systemic illness.